gs prelims–2018insightiasacademy.com/wp-content/uploads/upsc-prelims-2018-general... · (c)...

60
60/17, Above Subway, Old Rajinder Nagar,New Delhi-60 CENTRAL DELHI NORTH DELHI CONTACT B-18, Main Road, Satija House, Dr. Mukherjee Nagar, Delhi-09 011-45090051, 9818333201 INSIGHT IAS ACADEMY SOLUTIONS with DETAILED EXPLANATION [email protected] GS PRELIMS–2018 SET-B

Upload: others

Post on 17-Mar-2020

3 views

Category:

Documents


0 download

TRANSCRIPT

Page 1: GS PRELIMS–2018insightiasacademy.com/wp-content/uploads/UPSC-Prelims-2018-General... · (c) Bankim Chandra Chattopadhyay wrote Anandmath (d) Satyendranath Tagore became the first

60/17, Above Subway, Old Rajinder Nagar,New Delhi-60

CENTRAL DELHI NORTH DELHI CONTACTB-18, Main Road, Satija House, Dr. Mukherjee Nagar, Delhi-09

011-45090051, 9818333201

INSIGHT IAS ACADEMY

SOLUTIONS

with

DETAILED

EXPLANATION

[email protected]

GS PRELIMS–2018

SET-B

Page 2: GS PRELIMS–2018insightiasacademy.com/wp-content/uploads/UPSC-Prelims-2018-General... · (c) Bankim Chandra Chattopadhyay wrote Anandmath (d) Satyendranath Tagore became the first

Dear Aspirants,

Our team has categorised the questions along different levels of difficulties and

the relevance of the topics asked in the question paper so that you can understand

why was a particular type of question asked by UPSC. This will help you in

concentrating on most relevant sources and topics for your Mains Examination and

Prelims Examination next year if you need to face that. This analysis will also help

you in understanding that UPSC always picks up topics based on their applied

nature and significance.

In case you have any doubts or difficulty in any of the questions even after going

through this explanation booklet, you can meet the expert GS faculties of “INSIGHT

IAS Academy” in person at Old Rajinder Nagar Centre. Our team will be delighted to

help you always. The address and Contact Number of the INSIGHT Rajinder Nagar

Office are given on the cover page.

Wishing you all the best.

Prof. S. BALIYAN

Director

INSIGHT IAS ACADEMY

Congratulations! You have successfully appeared in IAS PRELIMS Examination

2018. Now it is our turn to help you in transforming your dreams into reality. We

understand that you are eager to find out your score in the GS PRELIMS 2018 Paper

as your success in Prelims examination depends only on the marks scored in this

question Paper. CSAT question paper is of qualifying nature and you need to score

only 33% in it. INSIGHT GS Team has prepared the answers of GS PRELIMS 2018

along with detailed explanation and sources. We are glad to say that our team has

been doing the same since last 10 years to help serious aspirants like you.

Page 3: GS PRELIMS–2018insightiasacademy.com/wp-content/uploads/UPSC-Prelims-2018-General... · (c) Bankim Chandra Chattopadhyay wrote Anandmath (d) Satyendranath Tagore became the first

INSIGHT GS PRELIMS 2018 EXPLANATIONS

Central Delhi: 60/17, Above Subway, Old Rajinder Nagar, Delhi - 60

North Delhi: B-18, Satija House, Main Road, Dr. Mukherjee Nagar, Delhi - 09

Ph: 011 - 45090051, 9818333201, 9871216382

SAME QUESTIONS IN ALL SETS

SET A SET B SET C SET D 1 41 86 16

2 42 87 17

3 43 88 18 4 44 89 19

5 45 90 20 6 46 91 21

7 47 92 22 8 48 93 23

9 49 94 24

10 50 95 25 11 51 96 26

12 52 97 27 13 53 98 28

14 54 99 29 15 55 100 30

16 86 71 61

17 87 72 62 18 88 73 63

19 89 74 64 20 90 75 65

21 91 76 66 22 92 77 67

23 93 78 68

24 94 79 69 25 95 80 70

26 96 81 71 27 97 82 72

28 98 83 73 29 99 84 74

30 100 85 75

31 56 11 86 32 57 12 87

33 58 13 88 34 59 14 89

35 60 15 90 36 61 16 91

37 62 17 92

38 63 18 93 39 64 19 94

40 65 20 95

Page 4: GS PRELIMS–2018insightiasacademy.com/wp-content/uploads/UPSC-Prelims-2018-General... · (c) Bankim Chandra Chattopadhyay wrote Anandmath (d) Satyendranath Tagore became the first

INSIGHT GS PRELIMS 2018 EXPLANATIONS

Central Delhi: 60/17, Above Subway, Old Rajinder Nagar, Delhi - 60

North Delhi: B-18, Satija House, Main Road, Dr. Mukherjee Nagar, Delhi - 09

Ph: 011 - 45090051, 9818333201, 9871216382

41 66 21 96

42 67 22 97 43 68 23 98

44 69 24 99 45 70 25 100

46 16 56 31 47 17 57 32

48 18 58 33

49 19 59 34 50 20 60 35

51 21 61 36 52 22 62 37

53 23 63 38 54 24 64 39

55 25 65 40

56 26 66 41 57 27 67 42

58 28 68 43 59 29 69 44

60 30 70 45 61 71 41 1

62 72 42 2

63 73 43 3 64 74 44 4

65 75 45 5 66 76 46 6

67 77 47 7 68 78 48 8

69 79 49 9

70 80 50 10 71 81 51 11

72 82 52 12 73 83 53 13

74 84 54 14 75 85 55 15

76 1 26 46

77 2 27 47 78 3 28 48

79 4 29 49 80 5 30 50

81 6 31 51 82 7 32 52

83 8 33 53

84 9 34 54

Page 5: GS PRELIMS–2018insightiasacademy.com/wp-content/uploads/UPSC-Prelims-2018-General... · (c) Bankim Chandra Chattopadhyay wrote Anandmath (d) Satyendranath Tagore became the first

INSIGHT GS PRELIMS 2018 EXPLANATIONS

Central Delhi: 60/17, Above Subway, Old Rajinder Nagar, Delhi - 60

North Delhi: B-18, Satija House, Main Road, Dr. Mukherjee Nagar, Delhi - 09

Ph: 011 - 45090051, 9818333201, 9871216382

85 10 35 55

86 11 36 56 87 12 37 57

88 13 38 58 89 14 39 59

90 15 40 60 91 31 1 76

92 32 2 77

93 33 3 78 94 34 4 79

95 35 5 80 96 36 6 81

97 37 7 82 98 38 8 83

99 39 9 84

100 40 10 85

Page 6: GS PRELIMS–2018insightiasacademy.com/wp-content/uploads/UPSC-Prelims-2018-General... · (c) Bankim Chandra Chattopadhyay wrote Anandmath (d) Satyendranath Tagore became the first

1

1. Which of the following to the introduction of English Education in India?

1. Charter Act of 1813

2. General Committee of Public Instruction

3. Orientalist and Anglicise Controversy

Select the correct answer using the code given below :

(a) 1 and 2 only

(b) 2 only

(c) 1 and 3 only

(d) 1, 2 and 3

Answer : (d)

Explanation :

Charter Act of 1813 asked Company to keep aside a sum of Rupiya One Lac per annum for progress of learning inIndia.

The General Committee of Public instruction was constituted on 17th July, 1823 by the acting Governor General Mr.Adam.

The forceful debate from 1823 to 1835 as regards the best mode of educating the Indians through the indigenouseducation of the country or the imported and the imposed Western system of education is generally known as theAnglicist- Orientalist Controversy.

Souruce : (http://shodhganga.inflibnet.ac.in/bitstream/10603/36381/7/07_chapter%201.pdf)

Covered in INSIGHT CLASS NOTES

2. Which one of the following is an artificial lake?

(a) Kodaikanal (Tamil Nadu)

(b) Kolleru (Andhra Pradesh)

(c) Nainital (Uttarakhand)

(d) Renuka (Himachal Pradesh)

Answer : (a)

Explanation :

Which one of the following is an artificial lake?

Kodaikanal artificial

The weather is ideal for walking or cycling in shorts and T-shirts, say the locals. And, there is no better path than the fivekm that skirts the 154-year-old, artificially-built lake. It is the most beautiful and expressive feature of Kodaikanal.There are many ways of doing the lake. Simply walk around it. Or, hire rowboats or pedal boats at the KodaikanalBoat Club and go boating at least for an hour to relax and look around.

Source : http://www.thehindu.com/life-and-style/travel/cycle-around-the-kodaikanal-lake/article181543.40.ece

Page 7: GS PRELIMS–2018insightiasacademy.com/wp-content/uploads/UPSC-Prelims-2018-General... · (c) Bankim Chandra Chattopadhyay wrote Anandmath (d) Satyendranath Tagore became the first

2

KoleruKolleru Lake is one of the largest freshwater lakes in India located in state of Andhra Pradesh and formsthe largest shallow freshwater[3.] lake in Asia, 15 kilometers away from the city of Eluru. Kolleru is locatedbetween Krishna and Godavari deltaNainital (uttarkhand)Source : https://en.wikipedia.org/wiki/Nainital_LakeRenuka (Himachal Pradesh)Renuka lake – (Largest natural lake in HP)Covered in INSIGHT PRINTED NOTES3. With reference to Pradhan Mantri Kaushal Vikas Yojana, consider the following statements

1. It is the flagship scheme of the Ministry of Labour and Employment.2. It, among other things, will also impart training in soft skills, entrepreneurship, financial and digital literacy.3. It aims to align the competencies of the unregulmed workforce of the count, to the National Skill Qualification

Framework.Which of the statements given above is/are correct?(a) 1 and 3 only(b) 2 only(c) 2 and 3 only(d) 1, 2 and 3

Answer : (c)Explanation :Pradhan Mantri Kaushal Vikas Yojana (PMKVY) is the flagship scheme of the Ministry of Skill Development &Entrepreneurship (MSDE). The objective of this Skill Certification Scheme is to enable a large number of Indianyouth to take up industry-relevant skill training that will help them in securing a better livelihood. Individuals with priorlearning experience or skills will also be assessed and certified under Recognition of Prior Learning (RPL).Key Components of the Scheme:1. Short Term Training

The Short Term Training imparted at PMKVY Training Centres (TCs) is expected to benefit candidates ofIndian nationality who are either school/college dropouts or unemployed. Apart from providing training accordingto the National Skills Qualification Framework (NSQF), TCs shall also impart training in Soft Skills,Entrepreneurship, Financial and Digital Literacy. Duration of the training varies per job role, rangingbetween 150 and 3.00 hours.

2. Recognition of Prior LearningIndividuals with prior learning experience or skills shall be assessed and certified under the Recognition of PriorLearning (RPL) component of the Scheme. RPL aims to align the competencies of the unregulatedworkforce of the country to the NSQF.

Source: http://www.skilldevelopment.gov.in/pmkvy.htmlCovered in INSIGHT TEST SERIES

Page 8: GS PRELIMS–2018insightiasacademy.com/wp-content/uploads/UPSC-Prelims-2018-General... · (c) Bankim Chandra Chattopadhyay wrote Anandmath (d) Satyendranath Tagore became the first

3

4. In 1920, which of the following changed its name to “Swarajya Sabha”?

(a) All India Home Rule League

(b) Hindu Mahasabha

(c) South Indian Liberal Federation

(d) The Servants of Indian Society

Answer : (a)

Explanation :

After failing to get a full grip on the Congress, he accepted the presidency of the All India Home Rule League at theend of April 1920, changing its name to Swarajya Sabha.

Source: Gandhi and Nationalism: The Path to Indian Independence By Simone Panter-Brick, page no. 92 (https://books.google.co.in/books?id=aIcBgAAQBAJ&pg=PA92&dq=All+india+home+rule+league+changed+name+to+swarajya+sabha+in+1920&hl=en&sa=X&ved=0ahUKEwjHmrG5iLfbAhWFTX0KHb Z2BqkQ6AEIJjAA#v=onepage&q=All%20india%20home%20rule%20league%20changed%20name%20to%20swarajya%20sabha%20in%201920&f=false)

Covered in INSIGHT PRINTED NOTES

5. Which among the following events happend earliest?

(a) Swami Dayanand established Arya Samaj

(b) Dinabandhu Mitra wrote Neeldarpan

(c) Bankim Chandra Chattopadhyay wrote Anandmath

(d) Satyendranath Tagore became the first Indian to succeed in the Indian Civil Services Examination

Answer : (b)

Explanation :

In 1875 Swami Dayanand Saraswati founded Arya Samaj. The early HQ was established at Bombay. Latter on theheadquarters was shifted at Lahore.

Souruce : (https://en.wikipedia.org/wiki/Arya_Samaj)

Dinabandhu Mitra expressed his sentiments in a play entitled Nil Darpan (the original Bengali play was written by him)and published anonymously from Dhaka in 1860.

Souruce : (https://en.wikipedia.org/wiki/Nil_Darpan)

Anandamath is a Bengali fiction, written by Bankim Chandra Chattopadhyay and published in 1882.

Souruce : (https://en.wikipedia.org/wiki/Anandamath)

Satyendranath was selected for the Indian Civil Service in June, 1863.

Source : (https://en.wikipedia.org/wiki/Satyendranath_Tagore)

Covered in INSIGHT CLASS NOTES

Page 9: GS PRELIMS–2018insightiasacademy.com/wp-content/uploads/UPSC-Prelims-2018-General... · (c) Bankim Chandra Chattopadhyay wrote Anandmath (d) Satyendranath Tagore became the first

4

6. Which of the following is/are the possible consequence/s of heavy sand mining in riverbeds?

1. Decreased salinity in the river

2. Pollution of groundwater

3. Lowering of the water-table

Select the correct answer using the code given below :

(a) 1 only

(b) 2 and 3 only

(c) 1 and 3 only

(d) 1, 2 and 3

Answer : (b)

Explanation :

Saline water intrusion takes along the coastal aquifers from the nearby sea and increases the salinity in river.

Covered in INSIGHT GS FOUNDATION CLASSES

7. With reference to agricultural soils, consider the following statements :

1. A high content of organic matter in soil drastically reduces its water holding capacity.

2. Soil does not play any role in the sulphur cycle.

3. Irrigation over a period of time can contribute to the salinisation of some agricultural lands.

Which of the statements given above is/are correct?

(a) 1 and 2 only

(b) 2 only

(c) 1 and 3 only

(d) 1, 2 and 3

Answer : (b)

Explanation :

A high content of organic matter in soil drastically reduces its water holding capacity

Source : The importance of soil organic matter : FAO

Soil does not play any role in the sulphur cycle

Irrigation over a period of time can contribute to the salinization of some agricultural lands.

Covered in INSIGHT GS FOUNDATION CLASSES

8. The Partnership for Action on Green Economy (PAGE), a UN mechanism m assist countries transition towardsgreener and more inclusive economies, emerged at

(a) The Earth Summit on Sustainable Development 2002, Johannesburg

(b) The United Nations Conference on Sustainable Development 2012, Rio de Janeiro

Page 10: GS PRELIMS–2018insightiasacademy.com/wp-content/uploads/UPSC-Prelims-2018-General... · (c) Bankim Chandra Chattopadhyay wrote Anandmath (d) Satyendranath Tagore became the first

5

(c) The United Nations Framework Convention on Climate Change 2015, Paris

(d) The World Sustainable Development Summit 2016, New Delhi

Answer : (b)

Explanation :

In 2012, Rio+20 (the United Nations Conference on Sustainable Development) was held in Brazil. Theconference’s outcome document entitled ‘The Future We Want’ was a call to action for governments, businessand the UN alike to support countries interested in transition to a green economy.

PAGE was created as the UN’s direct response to this call of action. Four UN organizations, including ILO, UNEnvironment, UNIDO and UNITAR initially joined forces to be able to provide countries with broad and tailoredsupport.

Source: http://www.un-page.org/about/who-are-we

9. “3D, printing” has applications in which the following ?

1. Preparation of confectionery items

2. Manufacture of bionic ears

3. Automotive industry

4. Reconstructive surgeries

5. Data processing technologies

Select the correct answer using the code given below:

(a) 1, 3 and 4 only

(b) 2, 3 and 5 only

(c) 1 and 4 only

(d) 1, 2, 3, 4 and 5

Answer : (d)

Explanation :

As a proof of concept, we generated a bionic ear via 3D printing of a cell-seeded hydrogel matrix in theprecise anatomic geometry of a human ear, along with an intertwined conducting polymer consisting ofinfused silver nanoparticles. This allowed for in vitro culturing of cartilage tissue around an inductive coil antenna inthe ear, which subsequently enables readout of inductively-coupled signals from cochlea-shaped electrodes. The printedear exhibits enhanced auditory sensing for radio frequency reception, and complementary left and right ears can listento stereo audio music. Overall, our approach suggests a means to intricately merge biologic and nanoelectronicfunctionalities via 3D printing.

Source : https://www.ncb1.nlm.nih.gov/pmc/articles/PMC3925752/

A manufacturer’s 3D printing operation could potentially generate terabytes of data, requiring IT organizationsto manage and maintain increasingly large data sets that design, manufacturing, and quality teams can easilyaccess and manipulate.

Source : http://deloitte.wsj.com/cio/2015/05/11/3d-printing-prepare-now-for-data-onslaught/

Covered in INSIGHT GS FOUNDATION CLASSES

Page 11: GS PRELIMS–2018insightiasacademy.com/wp-content/uploads/UPSC-Prelims-2018-General... · (c) Bankim Chandra Chattopadhyay wrote Anandmath (d) Satyendranath Tagore became the first

6

10. Consider the following statements :

1. The Barren Island volcano is an active volcano located-in the Indian territory.

2. Barren Island lies about 140 km east of Great Nicobar

3. The last time the Barren lJand volcano erupted was in 1991 and it has remained inactive since then.

Which of the statements given above is/are correct?

(a) 1 only

(b) 2 and 3 only

(c) 3 only

(d) 1 and 3

Answer : (a)

Explanation :

India’s only active volcano — the Barren Island volcano — in the Andaman and Nicobar Islands is again spewing lavaand ash, according to a team of scientists from the Goa-based National Institute of Oceanography (NIO).

Barren Island lies about 140 km east of Great Nicobar

The Barren Island, about 140 km from Port Blair.

India’s only active volcano — the Barren Island volcano — in the Andaman and Nicobar Islands is again spewing lavaand ash in February 2017 , according to a team of scientists from the Goa-based National Institute of Oceanography(NIO). Abhay Mudholkar and B. Nagender Nath from the NIO, who saw the eruptions from a mile off the mountain,noted that the volcano was erupting in small episodes lasting 5-10 minutes.

Source : http://www.thehindu.com/news/national/the-barren-islandvolcano-erupts-again/article17369862.ece

Covered in INSIGHT PRINTED NOTES

11. Why is a plant cailed Prosopis juliflora often mentioned in news ?

(a) Its extract is widely used in cosmetics.

(b) It tends to reduce the biodiversity in the area in which it grows

(c) Its extract is used in the pesticides.

(d) None of the above

Answer : (b)

Explanation :

The kikar or Prosopis juliflora was brought to Delhi from Mexico by the British more than a century ago. The exoticplant became invasive and wiped out most of the native plants and along with it the animals, which once usedto roam in the ridges. With its deep roots, it had also wreaked havoc on city’s groundwater.

Source : https://www.hindustantimes.com/delhi-news/delhi-indigenous-vines-to-be-used-to-kill-invasive-tree-species/story-6owi4Ep4JysSKOvui521jL.html

Covered in INSIGHT TEST SERIES

Page 12: GS PRELIMS–2018insightiasacademy.com/wp-content/uploads/UPSC-Prelims-2018-General... · (c) Bankim Chandra Chattopadhyay wrote Anandmath (d) Satyendranath Tagore became the first

7

12. Consider the following statements :1. Most of the world’s coral reefs are in tropical waters.2. More than one-third of the world’s coral reefs are located in the territories of Australia, Indonesia and

Philippines.3. Coral reefs host far more number of animal phyla than those hosted by tropical rainforests.Which of the statements given above is/are correct?(a) 1 and 2 only(b) 3 only(c) 1 and 3 only(d) 1, 2 and 3

Answer : (d)Explanation :Most of the world’s coral reefs are in tropical watersMore than one-third of the world’s coral reefs are located in the territories of Australia, Indonesia and PhilippinesSource : https://www.aljazeera.com/indepth/features/coral-reefs-important-dying-18012813.5520949.htmlMore than half of the earth’s coral reefs is distributed over only five countries: Indonesia, Australia, the Philippines,France (including its overseas territories) and Papua New Guinea.Coral reefs host far more number of animal phyla than those hosted by tropical rainforests.32 of the 34 recognised animal Phyla are found on coral reefs compared to 9 Phyla in tropical rainforestsSource : https://www.iucn.org/content/coral-reefs-facts-and-figuresCovered in INSIGHT PRINTED NOTES13. “Momentum for Change : Climate Neutral Now” is an initiative launch by

(a) The Intergovernmental Panel on Climate Change(b) The UNEP Secretariat(c) The UNFCCC Secretariat(d) The World Meteorological Organization

Answer : (c)Explanation :Climate Neutral Now is an initiative launched by the UN Climate Change in 2015, aiming at encouraging and supportingall levels of society to take climate action to achieve a climate neutral world by mid-century, as enshrined in the ParisAgreement adopted the same year.The initiative therefore invites companies, organizations, governments and citizens to work towards climate neutralityby reducing their climate footprint thanks to a simple 3.-steps method, whereby they shall:• Measure their greenhouse gas emissions;• Reduce them as much as possible; and• Compensate those which cannot be avoided by using UN certified emission reductions (CERs).Source : https://unfccc.int/climate-action/climate-neutral-now

Page 13: GS PRELIMS–2018insightiasacademy.com/wp-content/uploads/UPSC-Prelims-2018-General... · (c) Bankim Chandra Chattopadhyay wrote Anandmath (d) Satyendranath Tagore became the first

8

Spearheaded by the United Nations Climate Change Secretariat, Momentum for Change shines a light on themost innovative, scalable and replicable examples of what people around the world are doing to tackle climate change.

Source : https://momentum.unfccc.int/

14. With reference to educational institutions during colonial rule in India, consider the following pairs :

Institution Founder

1. Sanskrit College — William Benaras Jones

2. Calcutta Madarsa — Warren Hastings

3. Fort William — Arthur College Wellesley

Which of the pairs given above is/are correct ?

(a) 1 and 2

(b) 2 only

(c) 1 and 3

(d) 3 only

Answer : (b)

Explanation :

Warren Hastings, himself an intellectual, set up the Calcutta Madrasa in 1781 for the study and learning of Persian andArabic. In 1791 the efforts of Jonathan Duncan, the British Resident at Benares, bore fruit and a Sanskrit College wasopened at Benares.

Souruce : (BL GROVER MODERN India, PAGE NO. 257)

Lord Richard Colley Wellesley (NOT ARTHUR WELLESLEY) took the first step for training of Civil Servants whenhe founded the fort William College, in Calcutta in November 1800 where the Civil Servants of the Company were toreceive training in the literature, science and languages of India.

Souruce : (BL GROVER MODERN India, PAGE NO. 176)

Covered in INSIGHT CLASS NOTES

15. Consider the following pairs :

Regions sometimes mentioned in news Country

1. Catalmga — Spain

2. Crimea — Hungary

3. Mindanao — Philippines

4. Oromia — Nigeria

Which of the pairs given above are correctly matched ?

(a) 1, 2 and 3

(b) 3 and 4 only

(c) 1 and 3 only

(d) 2 and 4 only

Answer : (c)

Page 14: GS PRELIMS–2018insightiasacademy.com/wp-content/uploads/UPSC-Prelims-2018-General... · (c) Bankim Chandra Chattopadhyay wrote Anandmath (d) Satyendranath Tagore became the first

9

Explanation :

1. Catalonia - Autonomous community of Spain

2. Crimea - It is located south of the Ukrainian region

3. Mindanao - Island in the Philippines

4. Oromia - Oromia is one of the nine ethnically based regional states of Ethiopia

16. Which one of the following statements correctly describes the meaning of legal tender money?

(a) The money which is tendered in courts oflaw to defray the fee of legal cases

(b) The money which a creditor is under compulsion to accept in settleMent of his claims

(c) The bank money in the form of cheques, drafts, bills of exchange, etc.

(d) The metallic money in circulation in a country

Answer : (b)

Explanation :

Legal tender is any official medium of payment recognized by law that can be used to extinguish a public or privatedebt, or meet a financial obligation. The national currency is legal tender in practically every country. A creditor isobligated to accept legal tender toward repayment of a debt.

Source : https://www.investopedia.com/terms/l/legal-tender.asp

Covered in INSIGHT TEST SERIES

17. If a commodity is provided free to the public by the Government, then

(a) the opportunity cost is zero.

(b) the opportunity cost is ignored.

(c) the opportunity cost is transferred from the consumers of the product to the tax-paying public.

(d) the opportunity cost is transferred from the consumers of the product to the Government.

Answer : (c)

Covered in INSIGHT PRINTED NOTES

18. Increase in absolute and per capita real GNP do not connote a higher level of economic development, if

(a) industrial output fails to keep pace with agricultural output.

(b) agricultural output fails to keep pace with industrial output.

(c) poverty and unemployment increase.

(d) imports grow faster than exports.

Answer : (c)

Covered in INSIGHT ECONOMY CLASSES

19. Consider the following statements :

Human capital formation as a concept is better explained in terms of a process which enables

1. individuals of a country to accumulate more capital.

2. increasing the knowledge, skill levels and capacities of the people of the country.

Page 15: GS PRELIMS–2018insightiasacademy.com/wp-content/uploads/UPSC-Prelims-2018-General... · (c) Bankim Chandra Chattopadhyay wrote Anandmath (d) Satyendranath Tagore became the first

10

3. accumulation of tangible wealth.

4. accumulation of intangible wealth.

Which of the statements given above is/are correct?

(a) 1 and 2

(b) 2 only

(c) 2 and 4

(d) I, 3 and 4

Answer : (c)

Explanation :

According to the OECD, human capital is defined as:

“the knowledge, skills, competencies and other attributes embodied in individuals or groups of individuals acquiredduring their life and used to produce goods, services or ideas in market circumstances”.

Source: OECD

Source: NCERT XI – Indian Economic Development

Covered in INSIGHT ECONOMY CLASSES

20. Despite being a high saving economy, capital fortnation may not result in significant increase in output due to

(a) weak administrative machinery

(b) illiteracy

(c) high population density

(d) high capital-output ratio

Answer : (d)

21. After the Santhal Uprising subsided, what was/were the measure/measures taken by the colonial government?

1. The territories called ‘Santhal Paraganas’ were created.

2. It became illegal for a Santhal to transfer land to a non-Santhal.

Page 16: GS PRELIMS–2018insightiasacademy.com/wp-content/uploads/UPSC-Prelims-2018-General... · (c) Bankim Chandra Chattopadhyay wrote Anandmath (d) Satyendranath Tagore became the first

11

Select the correct answer using the code given below :

(a) 1 only

(b) 2 only

(c) Both 1 and 2

(d) Neither 1 nor 2

Answer : (c)

Explanation :

Source: https://www.academia.edu/3.5655119/Rethinking_Adivasi_Identity_The_Santhal_and_The_ Munda_RebellionCovered in INSIGHT CLASS NOTES22. Economically, one of the results of the Britith rule in India in the 19th century was the

(a) increase in the export of Indian handicrats(b) growth in the number of Indian owned factories(c) commercialization of Indian agriculture(d) rapid increase in the urban population

Answer : (c)Explanation :In the latter half of the 19th century another significant trend in Indian agriculture was the emergence of the commercializationof agriculture.Source : (BL GROVER MODERN India, PAGE NO. 446)Covered in INSIGHT CLASS NOTES23. If the President of India exercises his Power as provided under Article 3.56 of the Constitution in respect of a

particular State, then(a) the Assembly of the State is automatically dissolved.(b) the powers of the Legislature of that State shall be exercisable by or under the authority of the Parliament.(c) Article 19 is suspended in that State.

(d) the President can make laws relating to that State.

Answer : (b)

Page 17: GS PRELIMS–2018insightiasacademy.com/wp-content/uploads/UPSC-Prelims-2018-General... · (c) Bankim Chandra Chattopadhyay wrote Anandmath (d) Satyendranath Tagore became the first

12

Explanation :

Article 356. Provisions in case of failure of constitutional machinery in State

(1) If the President, on receipt of report from the Governor of the State or otherwise, is satisfied that asituation has arisen in which the government of the State cannot be carried on in accordance with theprovisions of this Constitution, the President may be Proclamation

(a) Assume to himself all or any of the functions of the Government of the State and all or any of the powers vestedin or exercisable by the Governor or anybody or authority in the State other than the Legislature of the State;

(b) Declare that the powers of the Legislature of the State shall be exercisable by or under the authorityof Parliament;

Source : M. Laxmikanth

Covered in INSIGHT POLITY CLASSES

24. Consider the following pairs :

Craft Heritage of

1. Puthukkuli shawls — Tamil Nadu

2. Sujni embroide — Maharashtra

3. Uppada Jamdani saris — Karnataka

Which of the pairs given above is/are correct?

(a) 1 only

(b) 1 and 2

(c) 3 only

(d) 2 and 3

Answer : (a)

Explanation :

Colourful shawls Toda tribals in Tamil Nadu. This shawl is referred to as pithukkuli because ‘putkuli’ or in theirnative language.

Source : (http://www.e-books-chennaimuseum.tn.gov.in/ChennaiMuseum/images/books/THE%20EMBROIDERED%20TEXTILES%20OF%20THE%20TODA%20TRIBES%20OF%20THE%20NILGIRIS.pdf)

Sujani (also known as Sujini) is a form of embroidery originating from the Bhusura village of Bihar in India.

Source : (https://www.utsavpedia.com/motifs-embroideries/sujani-embroidery/ )

Uppada Jamdani Sari is a silk sari style woven in Uppada of East Godavari district in the Indian state of AndhraPradesh.

Source : (https://en.wikipedia.org/wiki/Uppada_Jamdani_Sari)

Covered in INSIGHT PRINTED NOTES

25. In which of the following areas can GPS technology be used?

Page 18: GS PRELIMS–2018insightiasacademy.com/wp-content/uploads/UPSC-Prelims-2018-General... · (c) Bankim Chandra Chattopadhyay wrote Anandmath (d) Satyendranath Tagore became the first

13

1. Mobile phone operations2. Banking operations3. Controlling the power gridsSelect the correct answer using the code given below :(a) 1 only(b) 2 and 3 only(c) 1 and 3 only(d) 1, 2 and 3

Answer : (d)Explanation :There is an enormous, invisible clock that keeps ultra-precise time, can be checked from anywhere on earth, and is freefor everyone to use.This technological gift to mankind was built by the US government. It is called the Global Positioning System (GPS).Cell towers use it to route your phone calls, ATMs and cash registers use it for your transactions, electrical gridsuse it to send power to your house, and stock exchanges use it to regulate the trades that go into your stock portfolioor investment fund. And it is far more vulnerable to attack and disruption than most people know or are willing to admit.Source : https://qz.com/1106064/the-entire-global-financial-system-depends-on-gps-and-its-shockingly-vulnerable-to-attack/Covered in INSIGHT GS FOUNDATION CLASSES26. Consider the following statements :

1. The Reserve Bank of India manages and services Government of India Securities but not any StateGovernment Securities.

2. Treasury bills are issued by the Governrnent of India and there are no treasury bills issued by the StateGovernments.

3. Treasury bills offer are issued at a discount from the par value.Which of the statements given above is/are correct?(a) 1 and 2 only(b) 3. only(c) 2 and 3 only(d) 1, 2 and 3

Answer : (c)Explanation :What is a Government Security (G-Sec)?A Government Security (G-Sec) is a tradeable instrument issued by the Central Government or the State Governments.It acknowledges the Government’s debt obligation. Such securities are short term (usually called treasury bills, withoriginal maturities of less than one year) or long term (usually called Government bonds or dated securities with originalmaturity of one year or more). In India, the Central Government issues both, treasury bills and bonds or datedsecurities while the State Governments issue only bonds or dated securities, which are called the StateDevelopment Loans (SDLs).

Page 19: GS PRELIMS–2018insightiasacademy.com/wp-content/uploads/UPSC-Prelims-2018-General... · (c) Bankim Chandra Chattopadhyay wrote Anandmath (d) Satyendranath Tagore became the first

14

Treasury Bills (T-bills)

Treasury bills or T-bills, which are money market instruments, are short term debt instruments issued by the Governmentof India and are presently issued in three tenors, namely, 91 day, 182 day and 3.64 day. Treasury bills are zerocoupon securities and pay no interest. They are issued at a discount and redeemed at the face value atmaturity. For example, a 91 day Treasury bill of ¹ 100/- (face value) may be issued at say ¹ 98.20, that is, at a discountof say, ¹ 1.80 and would be redeemed at the face value of ¹ 100/-.How are the G-Secs issued?G-Secs are issued through auctions conducted by RB1. Auctions are conducted on the electronic platform calledthe E-Kuber, the Core Banking Solution (CBS) platform of RB1.Floatation of State Government Loans (State Development Loans)In terms of Sec. 21A (1) (b) of the Reserve Bank of India Act, 193.4, the RBI may, by agreement with any StateGovernment undertake the management of the public debt of that State. Accordingly, the RBI has entered intoagreements with 29 State Governments and one Union Territory (UT of Puducherry) for management oftheir public debt.Source: https://m.rb1.org.in/scripts/FAQView.aspx?Id=79

Covered in INSIGHT ECONOMY CLASSES27. Consider the following statements :

1. The Earth’s magnetic field has reversed every few hundred thousand years.2. When the Earth was created more than 4000 million years ago, there was 54% oxygen and no carbon

dioxide.3. When living organisms originated, they modified the early atmoshpere of the Earth.Which of the statements given above is/are correct?

(a) 1 only(b) 2 and 3 only

(c) 1 and 3 only

(d) 1, 2 and 3Answer : (c)

Explanation :

As a matter of geological record, the Earth’s magnetic field has undergone numerous reversals of polarity.Source : www.geomag.bgs.ac.uk/reversals.htmlThe Early atmosphere was probably dominated at first by water vapor, which, as the temperature dropped, wouldrain out and form the oceans. This would have been a deluge of truly global proportions an resulted in further reductionof CO2. Then the atmosphere was dominated by nitrogen, but there was certainly no oxygen in the early atmosphere.

Life started to have a major impact on the environment once photosynthetic organisms evolved. These organisms fedoff atmospheric carbon dioxide and converted much of it into marine sediments consisting of the innumerable shells anddecomposed remnants of sea creatures.

Source : https://globalchange.umich.edu/globalchange1/current/lectures/Perry_Samson_lectures/evolution_atm/

Page 20: GS PRELIMS–2018insightiasacademy.com/wp-content/uploads/UPSC-Prelims-2018-General... · (c) Bankim Chandra Chattopadhyay wrote Anandmath (d) Satyendranath Tagore became the first

15

28. The terms ‘WannaCry, Petya EternalBlue’ sometimes mentioned news recently are related to(a) Exoplanets(b) Cryptocurrency(c) Cyber attacks(d) Mini satellites

Answer : (c)Explanation :Source : https://www.theguardian.com/technology/2017/dec/3.0/wannacry-petya-notpetya-ransomwareCovered in INSIGHT GS FOUNDATION CLASSES29. With reference to the circumstances in Indian agriculture, the concept of “Conservation Agriculture” assumes

signifrcance” Which of the following fall under the Conservation Agriculture ?1. Avoiding the monoculture practices2. Adopting minimum tillage3. Avoiding the cultivation of plantation crops4. Using crop residues to cover soil surface5. Adopting spatial and temporal crop sequencing/crop rotationsSelect the correct answer using the code given below :(a) 1, 3 and 4(b) 2, 3, 4 and 5(c) 2, 4 and 5(d) 1, 2, 3 and 5

Answer : (c)

Explanation :

Source : http://www.fao.org/conservation-agriculture/en/

Page 21: GS PRELIMS–2018insightiasacademy.com/wp-content/uploads/UPSC-Prelims-2018-General... · (c) Bankim Chandra Chattopadhyay wrote Anandmath (d) Satyendranath Tagore became the first

16

30. The term “sixth mass extinction/sixth extinction” is often mentioned in the news in the context of the discussionof

(a) Widespread monoculture Practices agriculture and large-scale commercial farming with indiscriminate useof chemicals th many parts of the world that may result in the loss of good native ecosystems.

(b) Fears of a possible collision of a meteorite with the Earth in the near future in the manner it happened 65million years ago that caused the mass extinction of many species including those of dinosaurs.

(c) Large scale cultivation of genetically modified crops in many parts of the world and promoting their cultivationin other Parts of the world which may cause the disappearance of good native crop plan. and the loss offood biodiversity.

(d) Mankind’s over-exploitation/misuse of natural resources, fralitmentatiordloss , natural habitats, destructionof ecosystems, pollution and global climate change.

Answer : (d)

Explanation :

The scientists found billions of populations of mammals, birds, reptiles and amphibians have been lost all over theplanet, leading them to say a sixth mass extinction has already progressed further than was thought. Wildlife is dying outdue to habitat destruction, overhunting, toxic pollution, invasion by alien species and climate change. But the ultimatecause of all of these factors is “human overpopulation and continued population growth, and overconsumption

Source : https://www.theguardian.com/environment/2017/jul/10/earths-sixth-mass-extinction-event-already-underway-scientists-warn

31. Consider the following events

1. The first democratically elected communist party government formed in a State in India.

2. India’s then largest bank, ‘Imperial Bank of India’, was renamed ‘State Bank of India’.

3. Air India was nationalised and became the national carrier.

4. Goa became a part of independent India.

Which of the following is the correct chronological sequence of the above events?

(a) 4 - 1 - 2 - 3

(b) 3 - 2 - 1 - 4

(c) 4 - 2 - 1 - 3

(d) 3 - 1 - 2 - 4

Answer : (b)

Explanation :

First democratically elected communist party government formed in a state in India in 1957. The KeralaLegislative Assembly election of 1957 was the first assembly election in the Indian state of Kerala. The CommunistParty of India won the election with 60 seats.

Souruce : (https://en.wikipedia.org/wiki/Communist_Party_of_India)

Imperial bank was renamed as state bank of India in 1955. (https://en.wikipedia.org/wiki/

Page 22: GS PRELIMS–2018insightiasacademy.com/wp-content/uploads/UPSC-Prelims-2018-General... · (c) Bankim Chandra Chattopadhyay wrote Anandmath (d) Satyendranath Tagore became the first

17

Imperial_Bank_of_India).

Air India was nationalised in 1953. Government of India passed the Air Corporations Act and purchased a majoritystake in the carrier from Tata Sons.

Souruce : (https://en.wikipedia.org/wiki/Air_India#Nationalisation)

Goa became an independent India on 18 December 1961. Indian troops crossed the border into Goa and “liberated”it.

Souruce : (https://en.wikipedia.org/wiki/History_of_Goa#Decline).

Covered in INSIGHT PRINTED NOTES

32. Right to Privacy is protected as an intrinsic part of Right to Life and Personal Liberty. Which of the following inthe Constitution of India correctly and appropriately imply the above statement?

(a) Article 14 and the provisions under the 4th Amendment to the Constitution

(b) Article 17 and the Directive Principles of State Policy in Part IV

(c) Article 21 and the freedoms guaranteed in Part 3

(d) Article 24 and the provisions under the 44th Amendment to the Constitution

Answer : (c)

Explanation :

The right to privacy is protected as an intrinsic part of the right to life and personal liberty under Article 21 and as apart of the freedoms guaranteed by Part 3 of the Constitution.

Source: http://www.livelaw.in/supreme-courts-right-privacy-judgment-foundations/

Covered in INSIGHT POLITY CLASSES

33. Consider the following :

1. Areca nut

2. Barley

3. Coffee

4. Finger millet

5. Groundnut

6. Sesamum

7. Thrmeric

The Cabinet Committee on Economic Affairs h. announced the Minimum Support Price for which of theabove?

(a) 1, 2, 3 and 7 only

(b) 2, 4, 5 and 6 only

(c) 1, 3, 4, 5 and 6 only

(d) 1, 2, 3, 4, 5 and 7

Answer : (b)

Page 23: GS PRELIMS–2018insightiasacademy.com/wp-content/uploads/UPSC-Prelims-2018-General... · (c) Bankim Chandra Chattopadhyay wrote Anandmath (d) Satyendranath Tagore became the first

18

Explanation :

Source : http://agricoop.nic.in/sites/default/files/english.pdfCovered in INSIGHT ECONOMY CLASSES34. In which one of the following States is Pakhui Wildlife Sanctuary located ?

(a) Arunachal Pradesh(b) Manipur(c) Meghalaya

(d) NagalandAnswer : (a)Covered in INSIGHT PRINTED NOTES35. With reference to India’s satellite launch vehicl., consider the following statements :

1. PSLVs launch the satellites useful for Earth resources monitoring whereas GSLVs are designed mainly tolaunch communication satellites.

2. Satellites launched by PSLV appear to remain permanently fixed in the same position in the sky, as viewedfrom a particular location on Earth.

3. GSLV MK III is a four-staged launch vehicle with the first and third stages using solid rocket motors; andthe second and fourth stages using liquid rocket engines.

Which of the statements given above is/are correct ?

(a) 1 only(b) 2 and 3(c) 1 and 2(d) 3 only

Answer : (a)Explanation :PSLV is designed mainly to deliver the “earth-observation” or “remote-sensing” satellites with lift-off mass ofup to about 1750 Kg to Sun-Synchronous circular polar orbits of 600-900 Km altitude.The GSLV is designed mainly to deliver the communication-satellites to the highly elliptical (typically 250 x3.6000 Km) Geosynchronous Transfer Orbit (GTO). The satellite in GTO is further raised to its final destination, viz.,Geo-synchronous Earth orbit (GEO) of about 3.6000 Km altitude (and zero deg inclination on equatorial plane) byfiring its in-built on-board engines. Due to their geo-synchronous nature, the satellites in these orbits appear toremain permanently fixed in the same position in the sky, as viewed from a particular location on Earth, thusavoiding the need of a tracking ground antenna and hence are useful for the communication applications.Source : http://www.thehindu.com/sci-tech/science/what-is-the-difference-between-gslv-and-pslv/article6742299.eceGSLV Mk 3 is a three-stage heavy lift launch vehicle developed by ISRO. The vehicle has two solid strap-ons, acore liquid booster and a cryogenic upper stage.

Source : https://www.isro.gov.in/launchers/gslv-mk-3

Covered in INSIGHT GS PRELIMS CLASSES

Page 24: GS PRELIMS–2018insightiasacademy.com/wp-content/uploads/UPSC-Prelims-2018-General... · (c) Bankim Chandra Chattopadhyay wrote Anandmath (d) Satyendranath Tagore became the first

19

36. With reference to the governance of ‘public sector banking in India, consider the following statements:

1. Capital infusion into public sector banks by the Government of India has steadily increased in the lastdecade.

2. To put the public sector banks in order, the merger of associate banks with the parent State Bank of Indiahas been affected.

Which of the statements given above is/are correct

(a) 1 only

(b) 2 only

(c) Both 1 and 2

(d) Neither 1 nor 2

Answer : (b)

Explanation :

Source : https://rbi.org.in/scripts/PublicationsView.aspx?id=18060

Covered in INSIGHT GS FOUNDATION CLASSES

37. Consider the following items :

1. Cereal grains hulled

2. Chicken eggs cooked

3. Fish processed and canned

4. Newspapers containing advertising material

Page 25: GS PRELIMS–2018insightiasacademy.com/wp-content/uploads/UPSC-Prelims-2018-General... · (c) Bankim Chandra Chattopadhyay wrote Anandmath (d) Satyendranath Tagore became the first

20

Which of the above items is/are exempt under GST (Goods and Services Tax)?

(a) 1 only

(b) 2 and 3. only

(c) 1, 2 and 4 only

(d) 1, 2, 3. and 4

Answer : (c)

Explanation :

Previously GST council imposed 5% rate of GST on hulled grains. However, later on 11th June 2017, the rate of GSTapplicable on hulled grains is 0%. (nil rate) falls under GST HSN code number 1104.

Source : (http://howtoexportimport.com/Rate-of-GST-on-Hulled-cereal-grains-7241.aspx)

Chicken eggs cooked: 0% GST rate

Source : (https://www.exportgenius.in/gst-tax-rates/chicken-egg)

Fish processed and canned: 5% GST rate

Source : (https://www.indiafilings.com/learn/gst-rate-for-fish-fish-products-and-seafood/)

Newspapers containing advertising material: 0% GST rate

Source : (https://cleartax.in/s/printed-books-brochures-newspapers-gst-rate-hsn-code)

Covered in INSIGHT GS PRELIMS CLASSES

38. Consider the following statements :

1. The definition of “Critical Wildlife Habitat is incorporated in the Forest Rights Act, 2006.

2. For the first time in India, Baigas have been given Habitat Rights.

3. Union Ministry of Environment, Forest and Climate Change officially decides and declares Habitat Rightsfor Primitive and Vulnerable Tribal Groups in any part of India.

Which of the statements given above is/are correct?

(a) 1 and 2 only

(b) 1, 2 and 2 only

(c) 3 only

(d) 1, 2 and 3

Answer : (a)

Explanation :

The phrase ‘critical wildlife habitat’ is defined only in the Scheduled Tribes and Other Traditional Forest Dwellers(Recognition of Forest Rights) Act, 2006, and NOT in the Wildlife (Protection) Act, 1972.

Source : http://pib.nic.in/newsite/PrintRelease.aspx?relid=69806

Source : http://www.downtoearth.org.in/news/baiga-tribals-become-india-s-first-community-to-get-habitat-rights-52452

Page 26: GS PRELIMS–2018insightiasacademy.com/wp-content/uploads/UPSC-Prelims-2018-General... · (c) Bankim Chandra Chattopadhyay wrote Anandmath (d) Satyendranath Tagore became the first

21

Union Ministry of Tribal Affairs officially decides and declares Habitat Rights for Primitive and Vulnerable TribalGroups in any part of India.

39. Consider the followmg :

1. Birds

2. Dust blowing

3. Rain

4. Wind blowing

Which of the abow spread plant diseases?

(a) 1 and 3 only

(b) 3 and 4 only

(c) 1, 2 and 4 only

(d) I, 2, 3 and 4

Answer : (d)

Explanation :

Nematodes, snails, birds, and wild and domestic animals often help dissemination of plant diseases .The spores ofmany parasitic fungi are disse-minated by air currents from diseased to disease-free host Pathogens like, bacteria areoften disseminated by splashing of raindrops, as in case of Citrus canker disease.Soil and field operation also disseminatethe diseases as they result in dust blowing.

Source : http://www.biologydiscussion.com/plants/plant-diseases/dissemination-of-plant-diseases-16-mechanisms-botany/58671

40. With references to organic farming in India, consider the following statements :

1. The National Programme for Organic Production’ lNPOP) is operated under the guidelines and directionsof the Union blinistry of Rural Developnient.

2. ‘The Agricultural and Processed Food Products Export Development Authority’ (APED, functions theSecretariat for the implementation of NPOP.

3. Sikkim has beconte India’s first fully organic State.

Which of the statements given above is/are correct?

(a) 1 and 2 only

(b) 2 and 3 only

(c) 3 only

(d) 1, 2 and 3

Answer : (b)

Explanation :

Ministry of Commerce has implemented the National Programme for Organic Production (NPOP) since2001. The objectives of NPOP are as under:

Page 27: GS PRELIMS–2018insightiasacademy.com/wp-content/uploads/UPSC-Prelims-2018-General... · (c) Bankim Chandra Chattopadhyay wrote Anandmath (d) Satyendranath Tagore became the first

22

1. To provide the means of evaluation of certification programme for organic agriculture and products (includingwild harvest, aquaculture, livestock products) as per the approved criteria.

2. To accredit certification programmes of Certification Bodies seeking accreditation.

3. To facilitate certification of organic products in conformity with the prescribed standards.

4. To facilitate certification of organic products in conformity with the importing countries organic standards asper equivalence agreement between the two countries or as per importing country requirements.

5. To encourage the development of organic farming and organic processing.

Source : http://pib.nic.in/newsite/PrintRelease.aspx?relid=155000

Source : http://www.thehindu.com/news/national/Sikkim-becomes-India%E2%80%99s-first-organic-state/article13.999445.ece

Covered in INSIGHT TEST SERIES

41. Consider the following statements :

1 In the first Lok Sabha, the single largest party in the opposition was the Swatantra Party.

2. In the Lok Sabha, a “Leader of the Opposition” was recognised for the first time in 1969.

3. In the Lok Sabha, if a party does not have a minimum of 75 members, its leader cannot be recognised asthe Leader of the Opposition.

Which of the statements given above is/are correct?

(a) 1 and 3 only

(b) 2 only

(c) 2 and 3 only

(d) 1, 2 and 3

Answer : (b)

Explanation :

1. The communist party of India was the single largest opposition party in the First general Election. CPI won in16 seats.

Source : https://web.archive.org/web/20141008191615/http://ec1.nic.in/eci_main/StatisticalReports/LS_1951/VOL_1_51_LS.PDF

2. The leader of Opposition was officially recognized and received statutory recognition through the Salary andAllowances of Leaders of Opposition in Parliament Act, 1977.

Source : https://web.archive.org/web/20100116211914/http://mpa.nic.in/actopp.htm

3. In order to the receive the status of opposition party and Consequent Leader of Opposition Tag a party shouldsecure 10% of the total strength of the House 1.e. 55 seats in the Lok Sabha.

Source : http://www.thehindu.com/news/national/new-house-cannot-have-opposition-leader/article603.43.55.ece

Covered in INSIGHT GS POLITY CLASSES

Page 28: GS PRELIMS–2018insightiasacademy.com/wp-content/uploads/UPSC-Prelims-2018-General... · (c) Bankim Chandra Chattopadhyay wrote Anandmath (d) Satyendranath Tagore became the first

23

42. Which of the following leaf modifications occur(s) in the deser areas to inhibit water loss?1 Hard and waxy leaves2. Tiny leaves3. Thorns instead of leavesSelect the correct answer using the code given below:(a) 2 and 3 only(b) 2 only(c) 3 only(d) 1, 2 and 3

Answer : (d)Explanation :

Source : Goh Cheng leongCovered in INSIGHT CLASS NOTES43. As per the NSSO 70th Round “Situation Assessment Survey of Agriculture Household”, consider the following

statements :1. Rajasthan has the highest percentage share of agricultural household among its rural household.2. Out of the total agricultural housuhold in the country, a little over 60 percent belong to OBCs.3. In Kerala, a little over 60 percent of agricultuural household reported to have received maximum income

from sources other than agricultural activities.Which of the statements given above is/are correct?(a) 2 and 3 only(b) 2 only(c) 1 and 3 only(d) 1, 2 and 3

Answer : (c)

Page 29: GS PRELIMS–2018insightiasacademy.com/wp-content/uploads/UPSC-Prelims-2018-General... · (c) Bankim Chandra Chattopadhyay wrote Anandmath (d) Satyendranath Tagore became the first

24

Explantation :

The results of the survey are based on the Central Sample canvassed by NSSO, consisting of 4,529 villages spreadover rural areas of all States and Union Territories. A total number of 3.5,200 households were surveyed in first visitand 3.4,907 of them could be re-surveyed in second visit. Some salient findings of the survey regarding situation ofagricultural households in the country are as follows:A. Number of Agricultural households• During the agricultural year July 2012- June 2013., rural India had an estimated total of 90.2 million agricultural

households, which constituted about 57.8 percent of the total estimated rural households of the country duringthe same period.

• Uttar Pradesh, with an estimate of 18.05 million agricultural households, accounted for about 20 percent of allagricultural households in the country.

• Rajasthan had highest percentage of agricultural households (78.4 percent) among its rural householdsand Kerala had the least percentage share of agricultural households (27.3. percent) in its rural households.

• Out of the total estimated agricultural households in the country, about 45 percent belonged to Other BackwardClasses.

B. Source of Income of Agricultural households• Principal source of income of agricultural households is largely determined by the extent of land possession.• Majority of the agricultural households which possessed more than 0.40 hectare land, reported cultivation as

their principal source of income.• Among the agricultural households having less than 0.01 hectare land, about 56 percent reported wage/salary

employment as their principal source of income and another 23. percent reported livestock as their principalsource of income.

• Agricultural activity (cultivation, livestock and other agricultural activities) was reported to be the principalsource of income for majority of the agricultural households in all the major States, except Kerala whereabout 61 percent of the agricultural households reported to have received maximum income fromsources other than agricultural activities.

Source : NSSO 70th Round Report ‘Key Indicators of Situation of Agricultural Households in India’44. How is the National Green Tribunal (NGT) different from the Central Pollution Control Board (CPCB)?

1 The NGT has been established by an Act whereas the CPCB has been created by an exective order of theGoverment.

2. The NGT prodives environmental justice and helps reduce the burden of litigation in the higher courtswhereas the CPCB promotes cleanliness of streams and wells, and aims to improve the quality of air in thecountry.

Which of the statements given above is/are correct?(a) 1 only(b) 2 only(c) Both 1 and 2(d) Neither 1 nor 2

Answer : (b)

Page 30: GS PRELIMS–2018insightiasacademy.com/wp-content/uploads/UPSC-Prelims-2018-General... · (c) Bankim Chandra Chattopadhyay wrote Anandmath (d) Satyendranath Tagore became the first

25

Explanation :The National Green Tribunal has been established on 18.10.2010 under the National Green Tribunal Act 2010for effective and expeditious disposal of cases relating to environmental protection and conservation of forests andother natural resourcesSource : http://www.greentribunal.gov.in/The Central Pollution Control Board (CPCB), statutory organisation, was constituted in September, 1974 underthe Water (Prevention and Control of Pollution) Act, 1974.Principal Functions of the CPCB, as spelt out in the Water (Prevention and Control of Pollution) Act, 1974, and theAir (Prevention and Control of Pollution) Act, 1981, (i) to promote cleanliness of streams and wells in differentareas of the States by prevention, control and abatement of water pollution, and (2) to improve the quality ofair and to prevent, control or abate air pollution in the country.Source : http://cpcb.nic.in/Introduction/Covered in INSIGHT PRINTED NOTES45. Consider the following statements :

1. The parliament of India can place a particular law in the Ninth Schedule of the Constitution of India.2. The validity of a law placed in the Ninth Schedule cannot be examined by any court and no judgement can

be made on it. Which of the statements given below above is/are correct?

(a) 1 only(b) 2 only(c) Both 1 and 2(d) Neither 1 nor 2

Answer : (a)Explanation :1. The first amendment to the Indian Constitution added the Ninth Schedule to the constitution. It was introduced

by the Nehru Government to save laws from judicial scrutiny. Laws included under the Ninth Schedule areLand reforms, reservation, nationalisation of private properties etc. Parliament has amended the constitution toinclude various provisions under ninth schedule.For example, the 4th amendment inserted six acts to the 9th schedule. The 17th amendment added 44 moreacts. The 29th amendment brought in 2 acts from Kerala. The 3 4th amendment in 1974 added 20 more landtenure and land reforms laws enacted by the states.

2. The mandate of ninth schedule is to prevent judicial scrutiny but in a landmark ruling in IR Coelho versusState of Tamil Nadu, 2007, the Supreme Court of India ruled that all laws (including those in the NinthSchedule) would be open to Judicial Review if they violated the basic structure of the constitution. TheSupreme Court judgment laid that the laws placed under Ninth Schedule after April 24, 1973. shall be open tochallenge in court if they violated fundamental rights guaranteed under Article 14, 19, 20 and 21 of theConstitution.

Source : http://www.legalserviceindia.com/articles/nineth.htm

Covered in INSIGHT GS POLITY CLASSES

Page 31: GS PRELIMS–2018insightiasacademy.com/wp-content/uploads/UPSC-Prelims-2018-General... · (c) Bankim Chandra Chattopadhyay wrote Anandmath (d) Satyendranath Tagore became the first

26

46. Which of the following best describes the term “Merchant Discount Rate” sometimes seen in news?

(a) The incentive given by a bank to a merchant for accepting payments through debit cards pertaining to thatbank.

(b) The amount paid back by banks to their customers when they use debit cards for financial transactions forpurchasing goods or services.

(c) The charge to a merchant by a bank for accepting payments from his customers through the bank’s debitcards.

(d) The incentive given by the Government to merchants for promoting digital payments by their customersthrough Point of Sale (PoS) machine and debit cards.

Answer : (c)

Explanation :

MDR is a fee charged from a merchant by a bank for accepting payments from customers through creditand debit cards in their establishments.

MDR compensates the card issuing bank, the lender which puts the PoS terminal and payment gateways such asMastercard or Visa for their services.

MDR charges are usually shared in pre-agreed proportion between the bank and a merchant and is expressed inpercentage of transaction amount.

Source : https://economictimes.indiatimes.com/wealth/spend/what-is-merchant-discount-rate/articleshow/62390733cms

Covered in INSIGHT GS ECONOMY CLASSES

47. What is/are the consequence/consequences of a country becoming the member of the ‘Nuclear SuppliersGroup’?

1. It will have access to the latest and most efficient nuclear technologies.

2. It automatically becomes a member of “The Treaty on the Non-Proliferation of Nuclear Weapons (NPT)”?

Which of the statements given above is/are correct?

(a) 1 only

(b) 2 only

(c) Both 1 and 2

(d) Neither 1 nor 2

Answer : (a)

Explanation :

Membership of the NSG means: Access to technology for a range of uses from medicine to building nuclear powerplants for India from the NSG which is essentially a traders’ cartel. India has its own indigenously developed technologybut to get its hands on state of the art technology that countries within the NSG possess, it has to become part of thegroup.

Source : (https://www.livemint.com/Politics/GgIv7xn2DuFpDbsmGTcGpK/Why-a-NSG-membership-is-important-to-India.html)

Page 32: GS PRELIMS–2018insightiasacademy.com/wp-content/uploads/UPSC-Prelims-2018-General... · (c) Bankim Chandra Chattopadhyay wrote Anandmath (d) Satyendranath Tagore became the first

27

NPT not required for Nuclear Suppliers Group membership: India

Source : (http://indianexpress.com/article/india/india-news-india/npt-not-required-for-nuclear-suppliers-group-membership-india/)

Covered in INSIGHT CURRENT AFFAIRS CLASSES

48. With reference to India’s decision to levy an equalization tax of 6% on online advertisement services offered bynon-resident entities, which of the following statements is/are correct?1. It is introduced as a part of the Income Tax Act.2. Non-resident entities that offer advertisement services in India can claim a tax credit in their home country

under the “Double Taxation Avoidance Agreements”.Select the correct answer using the code given below :(a) 1 only(b) 2 only

(c) Both 1 and 2

(d) Neither 1 nor 2Answer : (d)Explanation :Equalisation Levy is a tax on business transaction for online marketing in which any Indian pays a sum of more than Rs.1 lakh to non-residents entities such Google and Facebook etc.Since Equalisation Levy is outside the scope of tax treaties entered into by India with other countries, the foreigncompany cannot claim a tax credit in its home country.

Source : (https://timesofindia.indiatimes.com/business/india-business/govt-may-put-off-expansion-of-equalisation-levys-range/articleshow/62596459.cms)Covered in INSIGHT ECONOMY CLASSES49. Consider the following statements :

1. The Fiscal Responsibility and Budget Management (FRBM) Review Committee Report has recommendeda debt to GDP ratio of 60% for the general (combined) government by 2023., comprising 40% for theCentral Government and 20% for the State Government.

2. The Central Government has domestic liabilities of 21% of GDP as compared to that of 49% of GDP ofthe State Government.

3. As per the Constitution of India, it is mandatory for a State to take the Central Government’s consent forraising any loan if the former owes any outstanding liabilities to the letter.

Which of the statements given above is/are correct?(a) 1 only(b) 2 and 3 only

(c) 1 and 3 only

(d) 1, 2 and 3

Answer : (c)

Page 33: GS PRELIMS–2018insightiasacademy.com/wp-content/uploads/UPSC-Prelims-2018-General... · (c) Bankim Chandra Chattopadhyay wrote Anandmath (d) Satyendranath Tagore became the first

28

Explanation :

Source : FRBM Review Committee Report

Art. 293. (3.) A State may not without the consent of the Government of India raise any loan if there is stilloutstanding any part of a loan which has been made to the State by the Government of India or by its predecessorGovernment, or in respect of which a guarantee has been given by the Government of India or by its predecessorGovernment

Source: https://indiankanoon.org/doc/13.15094/

Covered in INSIGHT TEST SERIES

50. Consider the following statements :

1. The qauntity of imported edible oils is more than the domestic production of edible oils in the last five years.

2. The Government does not impose any customs duty on all the imported edible oils as a special case.

Which of the statements given above is/are correct?

(a) 1 only

(b) 2 only

(c) Both 1 and 2

(d) Neither 1 nor 2

Answer : (a)

Explanation :

According to ICRA, India occupies a prominent position in the world oilseeds industry with contribution of around10% in worldwide production. But the demand of edible oils (extracted from oilseeds in addition to palm oil) issignificantly higher than the domestic production, leading to dependence on imports (60% of requirement). In FY2016India’s total edible oil demand stood at 24 mn tonnes out of which 9 mn tonnes was met from domesticproduction and 15 mn tonnes met from imports.

Source: https://economictimes.indiatimes.com/news/economy/agriculture/india-still-highly-dependent-on-edible-oil-imports-icra/articleshow/60360710.cms

Import duty on crude edible vegetable oils like groundnut oil, olive oil, cotton seed oil, safflower seed oil, saffola oil,coconut oil, palm kernel/babassu oil, linseed oil, maize corn oil, castor oil, sesame oil, other fixed vegetable fats and oilshas been raised from 12.5 per cent to 3.0 per cent.

Similarly, import duty on these refined edible vegetable oils has been raised from 20 per cent to 3.5 per cent, accordingto the Budget 2018 document.

Page 34: GS PRELIMS–2018insightiasacademy.com/wp-content/uploads/UPSC-Prelims-2018-General... · (c) Bankim Chandra Chattopadhyay wrote Anandmath (d) Satyendranath Tagore became the first

29

India imports about 14-15 million tonnes of edible oil annually to meet domestic demand. At present, the import dutyon crude soyabean oil is 3.0 per cent, crude sunflower oil 25 per cent and crude rapeseed oil 25 per cent.

Source : https://economictimes.indiatimes.com/news/economy/policy/government-raises-import-duty-on-cotton-seed-oil-olive-oil/articleshow/62743500.cms

Covered in INSIGHT GS FOUUNDATION CLASSES

51. He wrote biographies of Mazzini, Garibaldi, Shivaj1 and Shrikrishna; stayed in America for some time; andwas also elected to the Central Assembly. He was

(a) Aurobindo Ghosh

(b) Bipin Chandra Pal

(c) Lala Lajpat Rai

(d) Motilal Nehru

Answer : (c)

Explanation :

Source : Political Thinkers of Modern India: Lala Lajpat Rai, Volume 15 edited by Verinder Grover (Page No. 14)

Covered in INSIGHT CLASS NOTES

52. Consider the following gatemen.

1. Aadhaar card can be used as a proof of citizenship or domicile.

2. Once issued, Aadhaar number cannot be deactivated or omitted by the Issuing Authority.

Which of the statements given above is/are correct?

(a) 1 only

(b) 2 only

(c) Both 1 and 2

(d) Neither 1 nor 2

Answer : (d)

Explanation :

Aadhar Card is not a proof of citizenship, Calcutta High Court ruled on December 26, 2016. (http://www.india.com/news/india/aadhar-card-not-valid-proof-of-citizenship-rules-calcutta-high-court-1724765/)

Page 35: GS PRELIMS–2018insightiasacademy.com/wp-content/uploads/UPSC-Prelims-2018-General... · (c) Bankim Chandra Chattopadhyay wrote Anandmath (d) Satyendranath Tagore became the first

30

Your Aadhaar can be deactivated under various circumstances. An Aadhaar with mixed or anomalous biometricinformation or multiple names in single name is a good candidate for deactivation. Aadhaar can also get deactivatedupon non-usage of the same for three consecutive years.

Source : (https://www.businesstoday.in/current/policy/aadhaar-can-be-deactivated-for-several-reasons-know-how-to-activate-it-again/story/259855.html)

53. Which of the following has/have shrunk immensely/dried up in the recent past due to human activities?1. Aral Sea2. Black Sea3. Lake BaikalSelect the correct answer using the codes given below:(a) 1 only

(b) 2 and 3(c) 2 only

(d) 1 and 3Answer : (d)Explanation :Once the fourth largest lake in the world, Central Asia’s shrinking Aral Sea has reached a new low, thanks todecades-old water diversions for irrigation and a more recent drought. Satellite imagery released this week by NASAshows that the eastern basin of the freshwater body is now completely dry.

https://news.nationalgeographic.com/news/2014/10/141001-aral-sea-shrinking-drought-water-environment/NASA - https://earthobservatory.nasa.gov/IOTD/view.php?id=13.96Siberian emergency services remained in a state of red alert after the water level in Lake Baikal dropped to a criticallow point as the world’s largest lake continues to dry up.Source : https://themoscowtimes.com/news/siberia-on-red-alert-as-lake-baikal-continues-to-dry-up-45208

54. “Rule of Law Index” is released by which of the following ?(a) Amnesty International

(b) International Court of Justice(c) The Office of UN Commissioner for Human Rights

(d) World Justice ProjectAnswer : (d)Explanation :

The WJP Rule of Law Index® measures rule of law adherence in 113. countries and jurisdictions worldwide based onmore than 110,000 household and 3.,000 expert surveys. Featuring primary data, the WJP Rule of Law Index measurescountries’ rule of law performance across eight factors: Constraints on Government Powers, Absence of Corruption,Open Government, Fundamental Rights, Order and Security, Regulatory Enforcement, Civil Justice, and CriminalJustice.

Source : https://worldjusticeproject.org/our-work/wjp-rule-law-index/wjp-rule-law-index-2017%E2%80%93.2018

Page 36: GS PRELIMS–2018insightiasacademy.com/wp-content/uploads/UPSC-Prelims-2018-General... · (c) Bankim Chandra Chattopadhyay wrote Anandmath (d) Satyendranath Tagore became the first

31

55. Which one of the following links all the ATMs in India?

(a) Indian Banks’ Association

(b) National Securities Depository Limit

(c) National Payments Corporation of India

(d) Reserve Bank oflndia

Answer : (c)

Explanation :

National Financial Switch (NFS) is the largest network of shared automated teller machines (ATMs) in India. It wasdesigned, developed and deployed by the Institute for Development and Research in Banking Technology (IDRBT) in2004, with the goal of inter-connecting the ATMs in the country and facilitating convenience banking.

National Financial Switch (NFS) ATM network having 3.7 members and connecting about 50,000 ATMs was takenover by NPCI from Institute for Development and Research in Banking Technology (IDRBT) on December 14, 2009.Over the span of few years, NFS ATM network has grown many folds and is now the leading multilateral ATMnetwork in the country. As on 3.1st August’ 17, there were 941 members that includes 101 Direct, 776 Sub members,56 RRBs and 8 WLAOs using NFS network connected to more than 2.3.7 Lac ATM.

Source: https://www.npc1.org.in/product-overview/national-financial-switch-product-overview

Covered in INSIGHT GS CLASSES

56. Regarding Money Bill, which of the following statements is not correct ?

(a) A bill shall be deemed to be a Money Bill if it contains on, provisions relating to imposition, abolition,remission, alteration or regulation of any tax.

(b) A Money Bill has provisions for the custody of the Consolidated F’und of India or the Contingency Fundof India.

(c) A Money Bill is concerned with the appropriation of moneys out of the Contingency Fund of India.

(d) A Money Bill deals with the regulation of borrowing of money or giving of any guarantee by the Governmentof India.

Answer : (c)

Explanation :

Definition of “Money Bills”.-(l) a Bill shall be deemed to be a Money Bill if it contains only provisions dealing with all orany of the following matters:

(a) The imposition, abolition, remission, alteration or regulation of any tax

(b) The regulation of the borrowing of money or the giving of any guarantee by the Government ofIndia, or the amendment of the law with respect to any financial obligations undertaken or to beundertaken by the Government of India

(c) The custody of the Consolidated Fund or the Contingency Fund of India, the payment of moneys intoor the withdrawal of moneys from any such Fund

(d) The appropriation of moneys out of the Consolidated Fund of India(no Contingency fund)

Page 37: GS PRELIMS–2018insightiasacademy.com/wp-content/uploads/UPSC-Prelims-2018-General... · (c) Bankim Chandra Chattopadhyay wrote Anandmath (d) Satyendranath Tagore became the first

32

(e) The declaring of any expenditure to be expenditure charged on the Consolidated Fund of India or the increasingof the amount of any such expenditure

(f) The receipt of money on account of the Consolidated Fund of India or the public account of India or thecustody or Issue of such money or the audit of the accounts of the Union or of a State.

Source: The constitution of India P.M Bakshi, 12th Edition, Page no. 13.5Covered in INSIGHT POLITY CLASSES57. With reference to the election of the President of India, consider the following statements :

1. The value of the vote of each MLA varies from State to State.2. The value of the vote of MPs of the Lok Sabha is more than the value of the vote of MPs of the Rajya

Sabha.Which of the statement given above is/are correct?(a) 1 only(b) 2 only(c) Both 1 and 2(d) Neither 1 nor 2

Answer : (a)Explanation :ELECTORAL COLLEGE Under rule 40 of the Presidential and Vice-Presidential Elections Rules, 1974, the ElectionCommission is required to maintain a list of members of the Electoral College referred to in Article 54 with theiraddresses corrected up to date.The list will contain the names of elected members of the Rajya Sabha, Lok Sabha and the elected members of StateLegislative Assemblies and NCT of Delh1 and UT of Puducherry1. Value of a MLA vote:

Value of MLA vote = (Total Population of the state* )/ (No of constituencies in the state x 1000)ExPunjab Population: 1,3.5,51,060 & Total MLAs : 117.Value of each MLA vote = 1161,3.5,51,060 / (117*1000)Andhra Pradesh (Erstwhile):Population: 4,3.5,02,708 & Total MLAs : 294.Value of each MLA vote = 1484,3.5,02,708 / (294*1000) = 148 [ 147.9, rounded off to 148]Value of the UP MLA vote is the highest at 208, while Sikkim is the lowest with seven.

2. TOTAL VALUE OF VOTES OF 776 MEMBERS OF PARLIAMENT = 708 ×776 = 5, 49,408.Since the value of votes for MP are calculated using a single formula, Value of the vote remains the same unlikevalue of votes of MLA’s.

Source: http://ec1.nic.in/eci_main/ElectoralLaws/HandBooks/President_Election_08062017.pdf

Covered in INSIGHT CLASS NOTES

Page 38: GS PRELIMS–2018insightiasacademy.com/wp-content/uploads/UPSC-Prelims-2018-General... · (c) Bankim Chandra Chattopadhyay wrote Anandmath (d) Satyendranath Tagore became the first

33

58. In the Indian context, what is the implication of ratifying the ‘Additional Protocol’ with the ‘International AtomicEnergy Agency (IAEA)’?(a) The civilian nuclear reactors come under IAEA safeguards.(b) The military nuclear instaations come under the inspection of IAEA.(c) The country will have the privilege to buy uranium from the Nuclear Suppliers Group (NSG).(d) The country automatically becomes a member of the NSG.

Answer : (a)Explanation :An Additional Protocol (AP) to the Safeguards Agreement between the Government of India and the IAEA forthe Application of Safeguards to Civilian Nuclear Facilities entered into force on 25 July 2014.Source : (https://www.iaea.org/newscenter/news/indias-additional-protocol-enters-force)Covered in INSIGHT CURRENT AFFAIRS59. Consider the following counties :

1. Australia2. Canada3. China4. India5. Japan6. USAWhich of the above are among the ‘free-trade partners’ of ASEAN ?(a) 1, 2, 4 and 6(b) 3, 4, 5 and 6(c) 1, 3, 4 and 5(d) 2, 3, 4 and 6

Answer : (c)Explanation :Apart from the ASEAN Free Trade Area (AFTA) between ASEAN member states, the regional trade bloc has signedseveral FTAs with some of the major economies in the Asia-Pacific region. These include the ASEAN-Australia-NewZealand FTA (AANZFTA), the ASEAN-China FTA (ACFTA), the ASEAN-India FTA (AIFTA), the ASEAN-Korea FTA (AKFTA), and the ASEAN-Japan Comprehensive Economic Partnership (AJCEP).Source: https://www.aseanbriefing.com/news/2017/12/07/aseans-free-trade-agreements-an-overview.htmlCovered in INSIGHT CURRENT AFFAIRS CLASSES60. With reference to the ‘Global Alliance for Climate-Smart Agriculture (GACSA)’ which of the following statements

is/are correct ?1. GACSA is an outcome of the Climate Sumrnit held in Pats in 2015.

2. Membership of GACSA does not create any binding obligations.

3. India was instrumental in the creation of GACSA.

Page 39: GS PRELIMS–2018insightiasacademy.com/wp-content/uploads/UPSC-Prelims-2018-General... · (c) Bankim Chandra Chattopadhyay wrote Anandmath (d) Satyendranath Tagore became the first

34

Select the correct answer using the code given below :(a) 1 and 3 only(b) 2 only(c) 2 and 3 only(d) 1, 2 and 3

Answer : (b)Explanation :The concept of Climate-Smart Agriculture (CSA) was originally developed by FAO and officially presented and at theHague Conference on Agriculture, Food Security and Climate Change in 2010, through the paper “Climate-SmartAgriculture: Policies, Practices and Financing for Food Security, Adaptation and Mitigation”.In 2014 an alliance was set up with this issue as its focal point: the GASCA (Global Alliance for Climate-SmartAgriculture). GACSA is an inclusive, voluntary and action-oriented multi-stakeholder platform on Climate-Smart Agriculture(CSA).Its vision is to improve food security, nutrition and resilience in the face of climate change. GACSA aims to catalyse andhelp create transformational partnerships to encourage actions that reflect an integrated approach to the three pillars ofCSA. Membership in the Alliance does not create any binding obligations and each member individually determines thenature of its participation.Source : http://www.fao.org/gacsa/about/en/61. Which of the following is/are the aimiaims of “Digital India” Plan of the Government of India?

1. Formation of India’s own Internet companies like China did.2. Establish a policy framework to encourage overseas multinational corporations that collect Big Data to

build their large data centres within our national geographical boundaries.3. Connect many of our villages to the Internet and bring Wi-Fi to many of our schools, public places and

major tourist centresSelect the correct answer using the code given below :(a) 1 and 2 only(b) 3 only(c) 2 and 3 only(d) 1, 2 and 3

Answer : (b)Explanation :The Digital India programme is centred on three key vision areas:• Digital Infrastructure as a Core Utility to Every Citizen• Governance and Services on Demand• Digital Empowerment of Citizens

Covered in INSIGHT PRINTED NOTES

Page 40: GS PRELIMS–2018insightiasacademy.com/wp-content/uploads/UPSC-Prelims-2018-General... · (c) Bankim Chandra Chattopadhyay wrote Anandmath (d) Satyendranath Tagore became the first

35

62. Consider the following pairs

Towns sometimes mentioned in news Country

1. Aleppo — Syria

2. Kirkuk — Yemen

3. Mosul — Palestine

4. Mazar-i-sharif — Afghanistan

Which of the pairs given above are correctly matched?

(a) 1 and 2

(b) 1 and 4

(c) 2 and 3

(d) 3 and 4

Answer : (b)

Explanation :

Town UPSC Country

1 Aleppo Syria CORRECT

2 kirkuk Yemen INCORRECT is a city in Iraq

3. Mosul Palestine INCORRECT a major city in Iraq.

4 Mazar-i-sharif Afghanistan CORRECT

• Iraqi forces complete takeover of Kirkuk province after clashing with Kurds

• The Indian government’s announcement that the remains of 3.9 Indian workers, kidnapped four years ago bythe Islamic State, have been found near Mosul in Iraq, has brought a painful closure to the episode

Covered in INSIGHT GS FOUNDATION CLASSES

63. In the Federation established by The Government of India Act of 193.5, residuary powers were given to the

(a) Federal Legislature

(b) Governor General

(c) Provincial Legislature

(d) Provincial Governors

Answer : (b)

Explanation :

Residuary legislative powers were vested in the Governor-General in the matter of the enlistment of subjects either inthe Federal Legislative list or the Provincial Legislative list or the concurrent Legislative list.

Source: DD.Basu Introduction to the Constitution of India, 21st Edition Page no. 3.4.

Covered in INSIGHT CLASS NOTES

Page 41: GS PRELIMS–2018insightiasacademy.com/wp-content/uploads/UPSC-Prelims-2018-General... · (c) Bankim Chandra Chattopadhyay wrote Anandmath (d) Satyendranath Tagore became the first

36

64. Consider the following statements :

1. The Speaker of the Legislative Assembly shall vacate his/her office if he/she ceases to he a member of theAssembly.

2. Whenever the Legislative Assembly is dissolved, the Speaker shall vacate his/her office immediately.

Which of the statements given above is/are correct?

(a) 1 only

(b) 2 only

(c) Both 1 and 2

(d) Neither 1 nor 2

Answer : (a)

Explanation :

1. Article 179 deals with Vacation and resignation of, and removal from, the offices of Speaker and DeputySpeaker.

Member holding office as Speaker or Deputy Speaker of an Assembly

(a) Shall vacate his office if he ceases to be a member of the Assembly

(b) May at any time by writing under his hand addressed, if such member is the Speaker, to the DeputySpeaker, and if such member is the Deputy Speaker, to the Speaker, resign his office

(c) May be removed from his office by a resolution of the Assembly passed by a majority of all the thenmembers of the Assembly

2. The Speaker holds office from the date of her election till immediately before the first meeting of the Legislativeassembly after the dissolution of the one to which she was elected. She is eligible for re-election. On thedissolution of the Legislative assembly, although the Speaker ceases to be a member of the House, she doesnot vacate her office.

Source: The constitution of India P.M Bakshi, 12th Edition, Page no. 172-173.

Covered in INSIGHT CLASS NOTES

65. Which one of the following reflects the most appropriate relationship between law and liberty?

(a) If there are more laws, there is less liberty.

(b) If there are no laws, there is no liberty.

(c) If there is liberty, laws have to be made by the people.

(d) If laws are changed too often, liberty is in danger.

Answer : (b)

Explanation :

Locke: “where there is no law, there is no freedom”

In 1689, John Locke wrote that “the end of law is not to abolish or restrain, but to preserve and enlarge freedom.”

Source : https://inpropriapersona.com/articles/locke-where-there-is-no-law-there-is-no-freedom/

Page 42: GS PRELIMS–2018insightiasacademy.com/wp-content/uploads/UPSC-Prelims-2018-General... · (c) Bankim Chandra Chattopadhyay wrote Anandmath (d) Satyendranath Tagore became the first

37

66. Consider the following statements :

1. No criminal proceedings shall be instituted against the Governor of a State in any court during his term ofoffice

2. The emoluments and allowances of the Governor of a State shall not be diminished during his term ofoffice.

Which of the statements given above is/are correct?

(a) 1 only

(b) 2 only

(c) Both 1 and 2

(d) Neither 1 nor 2

Answer : (c)

Explanation :

Article 361 Protection of President and Governors.-

361(2): No criminal proceedings whatsoever shall be instituted or continued against the President, or theGovernor of a State, in any court during his term of office.

Source: The constitution of India P.M Bakshi, 12th Edition, Page no.3.23.

2. Article 158: Conditions of Governor’s office

Article 158(4): The emoluments and allowances of the Governor shall not be diminished during his term of office.

Source: The constitution of India P.M Bakshi, 12th Edition, Page no.162.

Covered in INSIGHT PRINTED NOTES

67. The well-known painting “Beni Thani” belongs to the

(a) Bundi school

(b) Jaipur school

(c) Kangra school

(d) Kishangarh school

Answer : (d)

Explanation :

Bani-Thani is one of the most well known painting from the small state of Kishangarh. Nihalchand’s fame rests onthis painting of Bani-Thani which is a highly stylised portrait of Radha.

Source: History of Indian Art By Kajal Kanjilal, PAGE: 23.

Covered in INSIGHT CLASS NOTES

68. What is “Terminal High Altitude Area Defense (THAAD)”, sometimes seen in the news?

(a) An Israeli radar system

(b) India’s indigenous anti-missile programme

Page 43: GS PRELIMS–2018insightiasacademy.com/wp-content/uploads/UPSC-Prelims-2018-General... · (c) Bankim Chandra Chattopadhyay wrote Anandmath (d) Satyendranath Tagore became the first

38

(c) An American anti-missile system

(d) A defence collaboration between Japan and South Korea

Answer : (c)

Explanation :

Terminal High Altitude Area Defense (THAAD) is a transportable system that intercepts ballistic missiles inside oroutside the atmosphere during their final, or terminal, phase of flight.THAAD uses a one-stage hit-to-kill interceptor todestroy incoming ballistic missile targets.

Source : http://www.thehindu.com/opinion/op-ed/tackling-pyongyang/article17668021.ece

Covered in INSIGHT CURRENT AFFAIRS CLASSES

69. With reference to cultural history of India, consider the following statements :

1. Most of the Tyagaraja Kritis are devotional songs in praise of Lord Krishna.

2. Tyagaraja created several new ragas.

3. Annamacharya and Tyagaraja are contemporaries.

4. Annamacharya ldrtanas are devotional songs in praise of Lord Venkateshwara.

Which of the statements given above are correct?

(a) 1 and 3 only

(b) 2 and 4 only

(c) 1, 2 and 3

(d) 2, 3 and 4

Answer : (b)

Explanation :

Kakarla Tyagabrahmam, colloquially known as Tyâgarâja and Tyagayya, was one of the greatest composers of Carnaticmusic or Indian classical music. He was a prolific composer and highly influential in the development of the SouthIndian classical music tradition. Tyagaraja composed thousands of devotional compositions, most of them in praiseof Lord Rama;

Source : (https://en.wikipedia.org/wiki/List_of_compositions_by_Tyagaraja)

Tallapâka Annamâcârya (or Annamayya) (22 May 1408 – 4 April 1503.) was a 15th-century Hindu saint. KakarlaTyagabrahmam (4 May 1767 – 6 January 1847) or Saint Tyagaraja lived in 18th-19th century. (https://en.wikipedia.org/wiki/Tyagaraja)

Ta77apâka Annamâcârya composed songs called sankirtanas in praise of the god Venkateswara, a form of Vishnu.

Source : (https://en.wikipedia.org/wiki/Annamacharya)

Covered in INSIGHT PRINTED NOTES

70. Which of the following are regarded as the main features of the “Rule of Law”?

1. Limitation of powers

2. Equality before law

Page 44: GS PRELIMS–2018insightiasacademy.com/wp-content/uploads/UPSC-Prelims-2018-General... · (c) Bankim Chandra Chattopadhyay wrote Anandmath (d) Satyendranath Tagore became the first

39

3. People’s responsibility to the Government

4. Liberty and civil rights

Select the correct answer using the code given below :

(a) 1 and 3 only

(b) 2 and 4 only

(c) 1, 2 and 4 only

(d) I, 2, 3 and 4

Answer : (c)

Explanation :

1. Limitations of power: Absence of arbitrary power ensures no man is punished except for a breach of law.

2. Equality before the law: equal subjection of all citizens to the ordinary law of the land administered by theordinary law courts.

3. Peoples responsibility to the Government does not come under the ambit of rule of law

4. Liberty and civil rights: The constitution is the result of the rights of the individual as defined and enforced bycourts of law, rather than constitution being the source of the individual rights. The centre piece of rule of law isLiberty and civil rights

Source: A V Dicey’s book The Law of the Constitution (1885)

Covered in INSIGHT GS FOUNDATION CLASSES

71. With reference to the Indian Regional Navigation Satellite System (IRNSS), consider the following statements:

1. IRNSS has three Satellite in geostationary and four satellites th geosynchronous orbits.

2. IRNSS covers entire India and about 5500 sq. km beyond its borders.

3. India will have its own satellite navigation system with full global coverage by the middle of 2019.

Which of the statements given above is/are correct?

(a) 1 only

(b) 1 and 2 only

(c) 2 and 3 only

(d) None

Answer : (a)

Explanation :

IRNSS is an independent regional navigation satellite system being developed by India. It is designed to provideaccurate position information service to users in India as well as the region extending up to 1500 km from its boundary,which is its primary service area.

The space segment consists of the IRNSS constellation of eight satellites, NavIC. Three satellites are located insuitable orbital slots in the geostationary orbit and the remaining four are located in geosynchronous orbitswith the required inclination and equatorial crossings in two different planes. All the satellites of the constellation areconfigured identically. The satellites are configured with I-1K Bus to be compatible for launch on-board PSLV.

Page 45: GS PRELIMS–2018insightiasacademy.com/wp-content/uploads/UPSC-Prelims-2018-General... · (c) Bankim Chandra Chattopadhyay wrote Anandmath (d) Satyendranath Tagore became the first

40

There is not a system for global coverage yet by ISRO

Source : https://www.isro.gov.in/irnss-programme

Covered in INSIGHT GS PRELIMS CLASSES

72. Consider the following phenomena :

1. Light is affected by gravity.

2. The Universe is constantly expanding.

3. Matter warps its surrounding space-time.

Whith of the above is/are the prediction/predictions of Albert Einstein’s General Theory of Relativity, oftendiscussed in media ?

(a) 1 and 2 only

(b) 3 only

(c) 1 and 3 only

(d) 1, 2 and 3

Answer : (d)

Explanation :

According to Einstein’s General Relativity Theory,light will be affected in the same way matter is affected by gravity.This is because under this theory, we should think of gravity not in terms of vector like forces, but as a consequence ofthe “shape” of the universe.

Source : http://www.physlink.com/education/askexperts/ae661.cfm

Einstein’s general theory of relativity. Relativity implies that space and time can stretch to vast dimensions from a tinystarting point

Source : /www.nationalgeographic.com/magazine/2005/05/einstein-relativity-cosmology-space-time-big-bang/

General relativity is Einstein’s law of gravity, his explanation of that fundamental force which holds us to the surfaceof the Earth. Gravity, Einstein asserted, is caused by a warping of space and time—or, in a language we physicistsprefer, by a warping of spacetime.

Source : https://www.its.caltech.edu/~kip/index.html/PubScans/VI-47.pdf

Covered in INSIGHT GS FOUNDATION CLASSES

73. With reference to the Genetically Modified mustard (GM mustard) developed in India, consider the followingstatements :

1. GM mustard has the genes of a soil bacterium that give the plant the property of pest-resistance to a widevariety of pests.

2. GM mustard has the genes that allow the plant cross-pollination and hybrithzation.

3. GM mustard has been developed jointly by the IAR1 and Punjab Agricultural University.

Page 46: GS PRELIMS–2018insightiasacademy.com/wp-content/uploads/UPSC-Prelims-2018-General... · (c) Bankim Chandra Chattopadhyay wrote Anandmath (d) Satyendranath Tagore became the first

41

Which of the statements given above is/are correct?

(a) 1 and 3 only(b) 2 only(c) 2 and 3. only(d) 1, 2 and 3.

Answer : (b)Explanation :GM Mustard DMH-11 has been developed by a team of scientists at Delhi University led by former vice-chancellorDeepak Pental under a government-funded project. In essence, it uses a system of genes from soil bacterium thatmakes mustard — generally a self pollinating plant — better suited to hybridisation than current methods.Source : http://www.thehindu.com/sci-tech/agriculture/GM-mustard-is-safe-says-technical-body/article14624827.eceCovered in INSIGHT GS PRELIMS CLASSES74. Consider the following pairs :

Terms sometimes seen in news Context / Topic1. Belle 2 experiment Artificial Intelligence2. Blockchain technology Digital Cryptocurrency3. CRISPR - Cas9 Particle PhysicsWhich of the pairs given above is/are correctly matched?(a) 1 and 3 only(b) 2 only(c) 2 and 3 only(d) 1, 2 and 3

Answer : (b)Explanation :Belle-2 experiment in Tsukuba, Japan, by The High Energy Accelerator Research Organisation (KEK) is designedto study violations of the Standard Model of particle physics.Source : http://www.thehindu.com/todays-paper/tp-national/global-collaboration-project-belle-2-moves-a-step-forward/article1793.9571.eceBlockchain technology is an invention that allows for the distribution of digital information and was initially devised forthe Bitcoin cryptocurrency.It is an incorruptible digital ledger of economic transactions that can be programmedto record not just financial transactions but virtually everything of value.”Source : http://www.binarytribune.com/bitcoin-guide/introduction-to-blockchain-technology/CRISPR technology is a simple yet powerful tool for editing genomes.CRISPRs are specialized stretches of DNA.The protein Cas9 (or “CRISPR-associated”) is an enzyme that acts like a pair of molecular scissors, capable of cuttingstrands of DNA. Its many potential applications include correcting genetic defects, treating and preventing the spreadof diseases and improving crops.

Source : https://www.livescience.com/58790-crispr-explained.html

Covered in INSIGHT GS FOUNDATION CLASSES

Page 47: GS PRELIMS–2018insightiasacademy.com/wp-content/uploads/UPSC-Prelims-2018-General... · (c) Bankim Chandra Chattopadhyay wrote Anandmath (d) Satyendranath Tagore became the first

42

75. Which of the following statementa best describes “carbon fertilization”?

(a) Increased plant growth due to increased concentration of carbon dioxide in the atmosphere

(b) Increased temperature of Earth due to increased concentration of carbon dioxide in the atmosphere

(c) Increased acidity of oceans as a result of increased concentration of carbon dioxide in the atmosphere

(d) Adaptation of all living beings on Earth to the climate change brought about by. the increased concentrationof carbon dioxide in the atmosphere

Answer : (a)

Explanation :

The carbon fertilization effect (CFE) is in principle simple: the larger amount of carbon dioxide in the atmosphere thathas resulted from rising anthropogenic emissions should help the growth of plants, which use carbon dioxide duringphotosynthesis.

Source : http://environmentalresearchweb.org/cws/article/news/543.47

Covered in INSIGHT PRINTED NOTES

76. When the alarm of your smartphone rings in the morning, you wake up and tap it to stop the alarm which causesyour geyser to be switched on automatically. The smart mirror in your bathroom shows the day’s weather andalso indicates the level of water in your overhead tank. After you take some groceries from your refrigerator thrmaking breakfast, it recognises the shortage of stock in it and places an order for the supply of fresh groceryitems. When You step out of your house and lock the door, all lights, fans, geysers and AC machines getswitched off’ automatically. On your way to office, your car warns you about traffic congestion ahead andsuggests an alternative route, and if you are late for a nieeting, it send. a message to your office accordingly.

In the context of enteming communication technologies, which one of tho following terms best applies to theabove scenario ?

(a) Border Gateway Protocol

(b) Internet of Things

(c) Internet Protocol

(d) Virtual Private Network

Answer : (b)

Explanation :

Internet of Things .IoT describes a world where just about anything can be connected and communicate in an intelligentfashion. In other words, with the internet of things, the physical world is becoming one big information system.

Source : https://www.techopedia.com/definition/28247/internet-of-things-iot

Covered in INSIGHT GS FOUNDATION CLASSES

77. With reference to solar power production in India, consider the following statements :

1. India is the third largest in the world in the manufacture of silicon wafers used in photovoltaic units.

2. The solar power tariffs are determined by the Solar Energy Corporation of India.

Page 48: GS PRELIMS–2018insightiasacademy.com/wp-content/uploads/UPSC-Prelims-2018-General... · (c) Bankim Chandra Chattopadhyay wrote Anandmath (d) Satyendranath Tagore became the first

43

Which of the statements given above is/are correct?

(a) 1 only

(b) 2 only

(c) Both 1 and 2

(d) Neither 1 nor 2

Answer : (d)

Explanation :

As entrusted by the Electricity Act, 2003 the Commission has the responsibility to discharge the following functions:-

Mandatory Functions:-

• to regulate the tariff of generating companies owned or controlled by the Central Government;.

• to regulate the tariff of generating companies other than those owned or controlled by the Central Governmentspecified in clause (a), if such generating companies enter into or otherwise have a composite scheme forgeneration and sale of electricity in more than one State;

• to regulate the inter-State transmission of electricity ;

• to determine tariff for inter-State transmission of electricity

CERC also determines solar tarrifs

78. The staple commodities of export by the English East India Company from Bengal in the middle of the 18th

century were

(a) Raw cotton, oil-seeds and opium

(b) Sugar, salt, zinc and lead

(c) Copper, silver, gold, spices and tea

(d) Cotton, silk, saltpetre and opium

Answer : (d)

Page 49: GS PRELIMS–2018insightiasacademy.com/wp-content/uploads/UPSC-Prelims-2018-General... · (c) Bankim Chandra Chattopadhyay wrote Anandmath (d) Satyendranath Tagore became the first

44

Explanation :

Indigo, cotton, silk, Saltpetre, sugar, gram and Opium were the major items exported by English company from Bengalin the middle of 18th century.

Source : The Cambridge Economic History of India, Volume 2, page number 254

Covered in INSIGHT CLASS NOTES

79. Which one of the following is a very significant asrmet of the Champaran Satyagraha?

(a) Active all-India participation of lawyers, students and women in the National Movement

(b) Active involvement of Dalit and Tribal communities of India in the National Movement

(c) Joining of peasant unrest to India’s National Movement

(d) Drastic decrease in the cultivation of plantation crops and commercial crops

Answer : (c)

Explanation :

Champaran Satyagraha would always remain as the starting point, the yoking, for the first time, of the peasant unrest tothe national movement.

Source : (http://www.thehindu.com/news/cities/Delhi/champaran-satyagraha-was-a-new-phase-in-freedom-fight/article18192945.ece)

Covered in INSIGHT CLASS NOTES

80. Who among the following were the founders of the “Hind Mazdoor Sabha” established in 1948

(a) B. Krislum Pillai, E.M.S. Namboodiripad and K.C. George

(b) Jayaprakash Narayan, Deen Dayal Upadhyay and M.N. Roy

(c) C.P. Ramaswamy lyer, K. Kamaraj and Veeresalingam Pantulu

(d) Ashok Mehta, T.S. Ramanulam and G.G. Mehta

Answer : (d)

Explanation :

The HMS was founded in Howrah in west bengal on 29 December 1948, by socialists, Forward Bloc followers andindependent unionists. Its founders included Basawon Singh (Sinha), Ashok Mehta, R.S. Ruikar, ManibenKara, Shibnath Banerjee, R.A. Khedgikar, T.S. Ramanujam, V.S. Mathur, G.G. Mehta. R.S. Ruikar was electedpresident and Ashok Mehta general secretary.

Source : (https://en.wikipedia.org/wiki/Hind_Mazdoor_Sabha)

Covered in INSIGHT PRINTED NOTES

81. With reference to the religious practices in India, the “Sthanakvasi” sect belongs to

(a) Buddhism

(b) Jainism

(c) Vaishnavism

(d) Shaivism

Answer : (b)

Page 50: GS PRELIMS–2018insightiasacademy.com/wp-content/uploads/UPSC-Prelims-2018-General... · (c) Bankim Chandra Chattopadhyay wrote Anandmath (d) Satyendranath Tagore became the first

45

Explanation :

Sthânakavâsî is a sect of Úvçtâmbara Jainism founded by a merchant named Lavaji in 1653. AD.

Source : (https://en.wikipedia.org/wiki/Sthânakavâsî)

Covered in INSIGHT CLASS NOTES

82. With reference to the cultural history of India. consider the following statements :

1. White marble was used in maldng Buland Darwaza and Khankah at Fatehpur Sikri.

2. Red sandstone and marble were used in making Bara lmambara and Rumi Darwaza at Lucknow.

Which of the statements given above is/are correct?

(a) 1 only

(b) 2 only

(c) Both 1 and 2

(d) Neither 1 nor 2

Answer : (d)

Explanation :

Buland Darwaza and Ibadat Khana (also known as Khankah or religious place) at Fatehpur Sikri is “the most iconicarchitectural accomplishment of Akbar’s reign. It incorporates almost all the essential features of Akbar’s architecturaltraditions: red sandstone, stone carvings, relief by inserting white marble, etc.”

Instead of stones and marble, brick and lime were used in the construction of Bara Imambara.

Source : http://www.thehindu.com/opinion/columns/buland-darwaza-and-rumi-darwaza-gateways-to-heaven/article19700177.ece

Covered in INSIGHT CLASS NOTES

83. Which one of the following foreign travellers elaborately discussed about diamonds and diamond mines ofIndia?

(a) Francois Bernier

(b) Jean Baptiste Tavernier

(c) Jean de Thevenot

(d) Abbe Barthel Carre

Answer : (b)

Explanation :

The earliest trustworthy account of Golconda diamond mines was by French jeweller, Jean Baptiste Tavernier.

Source : (http://www.thehindu.com/todays-paper/tp-features/tp-youngworld/saga-of-a-stone/article2267870.ece)

Covered in INSIGHT PRINTED NOTES

Page 51: GS PRELIMS–2018insightiasacademy.com/wp-content/uploads/UPSC-Prelims-2018-General... · (c) Bankim Chandra Chattopadhyay wrote Anandmath (d) Satyendranath Tagore became the first

46

84. With reference to Indian history, who among the following is a future Buddha, yet to come to save the world ?

(a) Avalokiteshvara

(b) Lokesvara

(c) Maitreya

(d) Padmapani

Answer : (c)

Explanation :

Maitreya will be the fifth and future Buddha.

Source : (https://en.wikipedia.org/wiki/Maitreya#Future_coming_of_Maitreya)

Covered in INSIGHT PRINTED NOTES

85. Which one of the following statements does not apply to the system of Subsidiary Alliance introduced by LordWellesley ?

(a) To maintain a large standing army at other’s expense

(b) To keep India safe from Napoleonic danger

(c) To secure a fixed income for the Company

(d) To establish British paramountcy over the Indian States

Answer : (c)

Explanation :

To secure a fixed income for the Company was not one of the objectives of Subsidiary Alliance System.

Source : (BL GROVER MODERN India, PAGE NO. 93)

Covered in INSIGHT CLASS NOTES

86. Consider the following statements :

1. Capital Adequacy Ratio (CAR) is the amount that banks have to maintain in the form of their own funds tooffset any loss that banks incur if the account-holders fail to repay dues.

2. CAR is decided by each individual bank.

Which of the statements given above is/are correct?

(a) 1 only

(b) 2 only

(c) Both 1 and 2

(d) Neither 1 nor 2

Answer : (a)

Explanation :

Capital Adequacy Ratio (CAR) is the ratio of a bank’s capital in relation to its risk weighted assets and currentliabilities. It is decided by central banks and bank regulators to prevent commercial banks from taking excessleverage and becoming insolvent in the process.

Source: https://economictimes.indiatimes.com/definition/capital-adequacy-ratio

Page 52: GS PRELIMS–2018insightiasacademy.com/wp-content/uploads/UPSC-Prelims-2018-General... · (c) Bankim Chandra Chattopadhyay wrote Anandmath (d) Satyendranath Tagore became the first

47

Banks are required to maintain a minimum Pillar 1 Capital to Risk-weighted Assets Ratio (CRAR) of 9% on an on-going basis (other than capital conservation buffer and countercyclical capital buffer etc.). The Reserve Bank will takeinto account the relevant risk factors and the internal capital adequacy assessments of each bank to ensure that thecapital held by a bank is commensurate with the bank’s overall risk profile. This would include, among others, theeffectiveness of the bank’s risk management systems in identifying, assessing / measuring, monitoring and managingvarious risks including interest rate risk in the banking book, liquidity risk, concentration risk and residual risk. Accordingly,the Reserve Bank will consider prescribing a higher level of minimum capital ratio for each bank under the Pillar 2framework on the basis of their respective risk profiles and their risk management systems. Further, in terms of the Pillar2 requirements, banks are expected to operate at a level well above the minimum requirement.

Source: https://rbidocs.rb1.org.in/rdocs/content/pdfs/58BS3.00685FL.pdfn

Covered in INSIGHT ECONOMY CLASSES

87. The identity platform ‘Aadhar’ provides open “Application Programming Interfaces (APIs)”. What does itimply?

1. It can be integrated into any electronic device.

2. Online authentication using iris is possible.

Which of the statements given above is/are correct?

(a) 1 only

(b) 2 only

(c) Both 1 and 2

(d) Neither 1 nor 2

Answer : (c)

Explanation :

Aadhaar provides open Application Programming Interfaces or APIs, which can be integrated easily into any electronicdevice. These APIs enable online authentication using a fingerprint or iris.

Source : (https://blogs.timesofindia.indiatimes.com/toi-edit-page/how-digital-will-save-india-unlike-china-indias-economic-revolution-will-come-from-services-not-manufacturing/)

88. Very recently, in which of the following countries have lakhs of people either suffered from severe famine/acutemalnutrition or died due to starvation caused by war/ethnic conflicts ?

(a) Angola and Zambia

(b) Morocco and Tunisia

(c) Venezuela and Colombia

(d) Yemen and South Sudan

Answer : (d)

Explanation :

In mid febraury the famine was declared.It’s not anatural disaster.It is one which is being caused by displacement,war.

Source : https://www.stuff.co.nz/world/africa/9083.8488/unicef-starving-to-death-in-south-sudan-as-the-world-watches

Page 53: GS PRELIMS–2018insightiasacademy.com/wp-content/uploads/UPSC-Prelims-2018-General... · (c) Bankim Chandra Chattopadhyay wrote Anandmath (d) Satyendranath Tagore became the first

48

Some three million people have been forced to flee their homes for safety since the conflict began in March 2017.Some end up in filty make shift camps like this one where virtually no basic services exist

Source : https://edition.cnn.com/2017/12/19/middleeast/yemen-intl/index.html

89. Regarding Wood’s Dispatch, which of the following statements are true?

1. Grants-in-Aid system was introduced.

2. Establishment of universities was recommended.

3. English as a medium of instruction at all levels of education was recommended.

Select the correct answer using the code given below :

(a) 1 and 2 only

(b) 2 and 3 only

(c) 1 and 3 only

(d) 1, 2 and 3

Answer : (a)

Explanation :

Main recommendations in Charles Wood’s Despatch

• As to the medium of instruction, it declared that for higher education English language was the most perfectmedium of education. It also emphasised the importance of the vernacular languages, for it was through themedium of the vernacular languages,’ that European knowledge could infilter to the masses.

• It proposed the setting up of vernacular primary schools’ in the villages at the lowest stage, followed by Anglo-Vernacular high schools and an affiiiated college at the district level. .

• It recommended a system of grants-in-aid to encourage and foster private enterprise in the field of education.This grants in-aid was conditional on the institutions employing qualified teachers and maintaining proper standardsof teaching.

• Universities on the model of the London University were proposed for Calcutta, Bombay and Madras.

Source : (BL GROVER MODERN India, PAGE NO. 259)

Covered in INSIGHT CLASS NOTES

90. With reference to the Parliament of India, , which of the following Parliamentary Committees scrutinizes andreports to the House whether the powers to make regulations, rules, sub-rules, by-lavvs, etc. conferred by theConstitution or delegated by the Parliament are being properly exercised by the Executive within the scope ofsuch delegation?

(a) Committee on Government Assurances

(b) Committee on Subordinate Legislation

(c) Rules Committee

(d) Business Advisory Committee

Answer : (b)

Page 54: GS PRELIMS–2018insightiasacademy.com/wp-content/uploads/UPSC-Prelims-2018-General... · (c) Bankim Chandra Chattopadhyay wrote Anandmath (d) Satyendranath Tagore became the first

49

Explanation :

COMMITTEE ON SUBORDINATE LEGISLATION

There shall be a Committee on Subordinate Legislation to scrutinize and report to the Council whether thepowers to make rules, regulations, bye-laws, schemes or other statutory instruments conferred by the Constitution ordelegated by Parliament have been properly exercised within such conferment or delegation. In both houses suchcommittees are constituted. It came into being in 1953.

Source : https://rajyasabha.nic.in/rsnew/committees/committ_sub_rules.asp

Covered in INSIGHT PRINTED NOTES

91. Consider the following statements

1. As per the Right to Education (RTE) Act to be eligible for appointment as a teacher in a State a personwould be required to possess the minimum qualification laid down by the concerned State Council ofTeacher Education.

2. As per the RTE Act, for teaching prim, classes, a candidate is required to pass a Teacher Eligibility Testconducted in accord. with the National Council of Teather Education guidelines.

3. In India, more than 90% of teacher education institutions are directly under the State Governments.

Which of the statement given above is/are correct?

(a) 1 and 2

(b) 2 only

(c) 1 and 3

(d) 3 only

Answer : (b)

Explanation :

The National Council of Teacher Education (NCTE), as the academic authority notified by the Central Government,under the Right of Children to Free and Compulsory Education (RTE), Act 2009, has laid down the minimum educational& professional qualifications for a person to be eligible for an appointment as a teacher for classes I-VIII, which areapplicable to all schools imparting elementary education, including the schools under the State Governmentsand to qualify under a Teacher Eligibility Test (TET).

Source : http://righttoeducation.in/norms-appointment-teachers

92. Consider the following pairs :

Tradition State

1. Chapchar Kut festival — Mizoram

2. Khongjom Parba ballad — Manipur

3. Thang-Ta dance — Sikkim

Page 55: GS PRELIMS–2018insightiasacademy.com/wp-content/uploads/UPSC-Prelims-2018-General... · (c) Bankim Chandra Chattopadhyay wrote Anandmath (d) Satyendranath Tagore became the first

50

Which of the pairs given above is/are correct?

(a) 1 only

(b) 1 and 2

(c) 3 only

(d) 2 and 3

Answer : (b)

Explanation :

The Chapchar Kut is a festival of Mizoram, India.

Source : (https://en.wikipedia.org/wiki/Chapchar_Kut)

Khongjom Parva is Traditional Manipuri music.

Source : (http://manipurtimes.com/reviving-the-dying-traditional-manipuri-music-khongjom-parva-padma-shri-awardee-nameirakpam-ibemni/)

Thang Ta is an ancient Manipuri Martial Art developed from the war environment of Manipur & created by theMeite1.

Source : (https://www.oknortheast.com/2010/12/thang-ta-martial-art-dance.html)

Covered in INSIGHT PRINTED NOTES

93. Consider the following statements :

1. The Food Safety and Standards Act, 2006 replaced the Prevention of Food Adulteration Act, 1954.

2. The Food Safety and Standards Authority of India (FSSAD is under the charge of Director Gene. ofHealth Services in the Union Ministry of Health and Family Welfare.

Which of the statements given above is/are correct?

(a) 1 only

(b) 2 only

(c) Both 1 and 2

(d) Neither 1 nor 2

Answer : (a)

Explanation :

The Food Safety and Standards Authority of India (FSSAI) is an autonomous body has been established underFood Safety and Standards , 2006 which consolidates various acts & orders that have hitherto handled food relatedissues in various Ministries and Departments. FSSAI has been created for laying down science based standards forarticles of food and to regulate their manufacture, storage, distribution, sale and import to ensure availability of safe andwholesome food for human consumption.

The Food and Safety Standard Act,2006 is a bucket for all the older laws, rules and regulations for food safety. TheFSS Act took 8 older acts into one umbrella including Prevention of Food Adulteration Act, 1954

Page 56: GS PRELIMS–2018insightiasacademy.com/wp-content/uploads/UPSC-Prelims-2018-General... · (c) Bankim Chandra Chattopadhyay wrote Anandmath (d) Satyendranath Tagore became the first

51

Ministry of Health & Family Welfare, Government of India is the Administrative Ministry for the implementationof FSSA1. The Chairperson and Chief Executive Officer of Food Safety and Standards Authority of India (FSSAI) isappointed by Government of India.The Chairperson is in the rank of Secretary to Government of India.It is not underthe charge of Director General of Health services

Source : http://www.fssa1.gov.in/home/fss-legislation/food-safety-and-standards-act.html

Covered in INSIGHT TEST SERIES

94. The term “two-state solution. is sometimes mentioned in the news in the context of the

(a) China

(b) Israel

(c) Iraq

(d) Yemen

Answer : (b)

Explanation :

Two-state solution only way to end Palestinian-Israeli conflict: Sisi

Egyptian President Abdel-Fattah al-Sisi has said establishing a Palestinian state through the two-state solution is theonly way to achieve stability, peace, prosperity and development in the region.

Source: http://www.thehindu.com/news/international/two-state-solution-only-way-to-end-palestinian-israeli-conflict-sisi/article22461546.ece

Covered in INSIGHT GS FOUNDATION CLASSES

95. With reference to the provisions made under the National Food Security Act, 2013, consider the followingstatements :

1. The families coming under the category of ‘below poverty line (BPL)’ only are elible to receive subsidiesfood grains.

2. The eldest woman in a household, of age 18 years or above, shall be the head of the household for thepurpose of issuance of a ration card.

3. Pregnant women and lactating mothers are entitled to a ‘take-home ration’ of 1600 calories per day duringpregnancy and for six months thereafter.

Which of the statement given above is/are correct?

(a) 1 and 2

(b) 2 only

(c) 1 and 3

(d) 3 only

Answer : (b)

Page 57: GS PRELIMS–2018insightiasacademy.com/wp-content/uploads/UPSC-Prelims-2018-General... · (c) Bankim Chandra Chattopadhyay wrote Anandmath (d) Satyendranath Tagore became the first

52

Explanation :

Salient Features of NFSA

• Coverage and entitlement under Targeted Public Distribution System (TPDS): Upto 75% of the ruralpopulation and 50% of the urban population will be covered under TPDS, with uniform entitlement of 5 kg perperson per month. Not restricted to only BPL families.

• Subsidised prices under TPDS and their revision: Foodgrains under TPDS will be made available atsubsidised prices of Rs. 3./2/1 per kg for rice, wheat and coarse grains for a period of three years from the dateof commencement of the Act.

• Maternity Benefit: Pregnant women and lactating mothers will also be entitled to receive maternity benefitof not less than Rs. 6,000.

• Women Empowerment: Eldest woman of the household of age 18 years or above to be the head of thehousehold for the purpose of issuing of ration cards.

Source : http://dfpd.nic.in/Salient-features-National-Food-Security-Act.htm

Covered in INSIGHT TEST SERIES

96. India enacted The Geographical Indications of Goods (Registration and Protection) Act, 1999 in order tocomply with the obligations

(a) ILO

(b) IMF

(c) UNCTAD

(d) WTO

Answer : (d)

Explanation :

Article 22 of the TRIPS Agreements (WTO Agreement) defines Geographical Indication as “indications which identifya good as originating in the territory of a Member, or a region or locality in that territory, where a given quality,reputation or other characteristic of the good is essentially attributable to its geographical origin”.

Consequently, upon India joining as a member state of the TRIPS Agreement sui-geneis legislation for theprotection of geographical indication was enacted in 1999. Geographical Indications are covered as an element ofIntellectual Property Rights under the TRIPS Agreement (WTO Agreement). The object of the Geographical Indicationsof Goods (Registration and Protection) Act, 1999, is three fold,

• By specific law governing the geographical indication of goods in the country which could adequately protectthe interest of producers of such goods,

• To exclude unauthorized persons from misusing geographical indications and to protect consumers from deceptionand,

• To promote goods bearing Indian Geographical Indication in the export market.

Source : http://www.mondaq.com/india/x/688456/Trademark/Protection+Of+Geographical+Indications

http://pib.nic.in/newsite/PrintRelease.aspx?relid=1553.21

Covered in INSIGHT ECONOMY CLASSES

Page 58: GS PRELIMS–2018insightiasacademy.com/wp-content/uploads/UPSC-Prelims-2018-General... · (c) Bankim Chandra Chattopadhyay wrote Anandmath (d) Satyendranath Tagore became the first

53

97. Consider the following statements

1. In India, State Governments do not have the power to auction non-coal mines.

2. Andhra Pradesh and Jharkhand do not have gold mines.

3. Rajasthan has iron ore mines.

Which of the statements given above is/are correct?

(a) 1 and 2

(b) 2 only

(c) 1 and 3

(d) 3 only

Answer : (d)

Explanation :

Even though the state governments have been able to auction just 29 non-coal mines in the last 20 months, the Centralgovernment is now expecting them to auction 54 mines in the current financial year. Unlike coal, the auction of mininglicences of non-coal minerals is conducted by the respective state governments. The central government expectsthe state governments to earn total Rs 1.22 lakh crore, over the period of next 50 years, from the 29 mines alreadyauctioned.

Source : http://indianexpress.com/article/business/govt-states-likely-to-auction-54-non-coal-mines-in-fy18-4853.479/

With only three of its gold mines working (Hutt1 and Uti in Karnataka and the Hirabuddini mines in Jharkand) andproducing about 0.5% of the country’s annual gold consumption , India’s dependence on imports has increased.

Andhra Pradesh is literally a gold mine! The state has a huge potential for gold mining and is the only state in Indiawhere two exclusive gold mines are being set up by an Australian-Indian organisation.

Sources: https://timesofindia.indiatimes.com/city/visakhapatnam/ap-has-huge-gold-mining-potential/articleshow/56840679.cms

Source : https://timesofindia.indiatimes.com/city/mumbai/India-has-only-3.-working-gold-mines/articleshow/10799590.cms

SAIL may get a slice in Rajasthan iron ore mine

Source : https://economictimes.indiatimes.com/industry/indl-goods/svs/metals-mining/sail-may-get-a-slice-in-rajasthan-iron-ore-mine-source/articleshow/12965543.cms

Covered in INSIGHT PRINTED NOTES

98. With reference to digital payments, consider the following statements :

1. BHIM app allows the user to transfer money th anyone with a UPI-enabled bank account.

2. While a chippin debit card has four factors of authentication, BHIM app has only two factors ofauthentication.

Page 59: GS PRELIMS–2018insightiasacademy.com/wp-content/uploads/UPSC-Prelims-2018-General... · (c) Bankim Chandra Chattopadhyay wrote Anandmath (d) Satyendranath Tagore became the first

54

Which of the statements given above is/are correct?

(a) 1 only

(b) 2 only

(c) Both 1 and 2

(d) Neither 1 nor 2

Answer : (a)

Explanation :

Source : http://indianexpress.com/article/technology/tech-news-technology/bhim-app-narendra-modi-upi-online-transaction-safety-4453599/

99. Among the following cities, which one lies on a longitude closest to that of Delhi?

(a) Bengaluru

(b) Hyderabad

(c) Nagpur

(d) Pune

Answer : (a)

Explanation :

DELHI - 28.7041° N, 77.1025° E

Bengalure - 12.9716° N, 77.5946° E

Hyderbad - 17.3.850° N, 78.4867° E

Nagpur - 21.1458° N, 79.0882° E

Pune - 18.5204° N, 73.8567° E

Covered in INSIGHT PRINTED NOTES

100. International Labour Organisation, Conventions 138 and 182 are related to

(a) Child labour

(b) Adaptation of agricultural practices to global climate change

(c) Regulation of food prices and food security

(d) Gender parity at the workplace

Answer : (a)

Page 60: GS PRELIMS–2018insightiasacademy.com/wp-content/uploads/UPSC-Prelims-2018-General... · (c) Bankim Chandra Chattopadhyay wrote Anandmath (d) Satyendranath Tagore became the first

55

Explanation :

The Union Cabinet chaired by the Prime Minister Shri Narendra Modi has given its approval for ratification of the twofundamental conventions of the International Labour Organization namely, Minimum Age Convention (No 138)concerning minimum age for admission to employment and the Worst Forms of Child Labour Convention(No 182) concerning the prohibition and immediate action for elimination of the worst forms of Child Labour.

Source: http://pib.nic.in/newsite/PrintRelease.aspx?relid=1603.69

79 Questions out of 100 havecome from INSIGHT GS Classes,

Printed Material & Test Series